ANSWER SHEET
Practice Test 1
Section 1: Reading
Section 2: Writing
Section 3: Math (No Calculator)
Section 4: Math (Calculator)
Essay
PLANNING PAGE
READING TEST
65 MINUTES, 52 QUESTIONS
Turn to Section 1 of your answer sheet to answer the questions in this section.
Directions: Following each of the passages (or pairs of passages) below are questions about the passage (or passages). Read each passage carefully. Then, select the best answer for each question based on what is stated in the passage (or passages) and in any graphics that may accompany the passage.
Questions 1–11 are based on the following passage.
In this adaptation of an excerpt from An Occurrence at Owl Creek Bridge, a short story set in Civil War times, a man is about to be hanged. The first two paragraphs set the scene; the remainder of the passage presents a flashback to an earlier, critical encounter.
Line (5) (10) (15) (20) |
A man stood upon a railroad bridge in Northern Alabama, looking down into the swift waters twenty feet below. The man’s hands were behind his back, the wrists bound with a cord. A rope loosely encircled his neck. It was attached to a stout cross-timber above his head, and the slack fell to the level of his knees. Some loose boards laid upon the sleepers supporting the metals of the railway supplied a footing for him and his executioners—two private soldiers of the Federal army, directed by a sergeant, who in civil life may have been a deputy sheriff. At a short remove upon the same temporary platform was an officer in the uniform of his rank, armed. He was a captain. A sentinel at each end of the bridge stood with his rifle in the position known as “support”—a formal and unnatural position, enforcing an erect carriage of the body. It did not appear to be the duty of these two men to know what was occurring at the center of the bridge; they merely blockaded the two ends of the foot plank which traversed it. |
(25) (30) (35) (40) |
The man who was engaged in being hanged was apparently about thirty-five years of age. He was a civilian, if one might judge from his dress, which was that of a planter. His features were good—a straight nose, firm mouth, broad forehead, from which his long, dark hair was combed straight back, falling behind his ears to the collar of his well-fitting frock coat. He wore a moustache and pointed beard, but no whiskers; his eyes were large and dark grey and had a kindly expression that one would hardly have expected in one whose neck was in the hemp. Evidently this was no vulgar assassin. The liberal military code makes provision for hanging many kinds of people, and gentlemen are not excluded. |
(45) (50) (55) (60) |
Peyton Farquhar was a well-to-do planter, of an old and highly respected Alabama family. Being a slave-owner, and, like other slave-owners, a politician, he was naturally an original secessionist and ardently devoted to the Southern cause. Circumstances had prevented him from taking service with the gallant army that had fought the disastrous campaigns ending with the fall of Corinth, and he chafed under the inglorious restraint, longing for the release of his energies, the larger life of the soldier, the opportunity for distinction. That opportunity, he felt, would come, as it comes to all in war time. Meanwhile, he did what he could. No service was too humble for him to perform in aid of the South, no adventure too perilous for him to undertake if consistent with the character of a civilian who was at heart a soldier, and who in good faith and without too much qualification assented to at least a part of the frankly villainous dictum that all is fair in love and war. |
(65) (70) |
One evening while Farquhar and his wife were sitting near the entrance to his grounds, a grey-clad soldier rode up to the gate and asked for a drink of water. Mrs. Farquhar was only too happy to serve him with her own white hands. While she was gone to fetch the water, her husband approached the dusty horseman and inquired eagerly for news from the front. |
(75) (80) |
“The Yanks are repairing the railroads,” said the man, “and are getting ready for another advance. They have reached the Owl Creek bridge, put it in order, and built a stockade on the other bank. The commandant has issued an order, which is posted everywhere, declaring that any civilian caught interfering with the railroad, its bridges, tunnels, or trains, will be summarily hanged. I saw the order.” |
(85) | “How far is it to the Owl Creek bridge?” Farquhar asked. |
“About thirty miles.”
“Is there no force on this side of the creek?”
(90) | “Only a picket post half a mile out, on the railroad, and a single sentinel at this end of the bridge.” |
(95) | “Suppose a man—a civilian and a student of hanging—should elude the picket post and perhaps get the better of the sentinel,” said Farquhar, smiling, “what could he accomplish?” |
(100) | The soldier reflected. “I was there a month ago,” he replied. “I observed that the flood of last winter had lodged a great quantity of driftwood against the wooden pier at the end of the bridge. It is now dry and would burn like tow.” |
(105) | The lady had now brought the water, which the soldier drank. He thanked her ceremoniously, bowed to her husband, and rode away. An hour later, after nightfall, he repassed the plantation, going northward in the direction from which he had come. He was a Yankee scout. |
1. As used in line 13, “civil” most nearly means
(A) polite.
(B) noncriminal.
(C) nonmilitary.
(D) individual.
2. In cinematic terms, the first two paragraphs most nearly resemble
(A) a wide-angle shot followed by a close-up.
(B) a sequence of cameo appearances.
(C) a trailer advertising a feature film.
(D) two episodes of an ongoing serial.
3. It can most reasonably be inferred from the passage that the man awaiting hanging was
(A) innocent of any criminal intent.
(B) an unlikely candidate for execution.
(C) a victim of mistaken identity.
(D) purposely assuming a harmless demeanor.
4. Which choice provides the best evidence for the answer to the previous question?
(A) Lines 25–27 (“The man . . . age”)
(B) Lines 27–29 (“He was . . . planter”)
(C) Lines 33–38 (“He wore . . . hemp”)
(D) Lines 44–47 (“Being . . . cause”)
5. The author’s tone in discussing “the liberal military code” (line 39) can best be described as
(A) approving.
(B) ironic.
(C) irked.
(D) regretful.
6. It can most reasonably be inferred from the passage that Peyton Farquhar would consider which of the following a good example of how a citizen should behave in wartime?
(A) He should use even underhanded methods to support his cause.
(B) He should enlist in the army without delay.
(C) He should turn to politics as a means of enforcing his will.
(D) He should avoid involving himself in disastrous campaigns.
7. As used in line 59, “consistent” most nearly means
(A) unchanging.
(B) compatible.
(C) logically sound.
(D) steady and predictable.
8. It can most reasonably be inferred from the passage that Mrs. Farquhar is
(A) sympathetic to the Confederate cause.
(B) too proud to perform menial tasks.
(C) uninterested in news of the war.
(D) reluctant to ask her slaves to fetch water.
9. Which choice provides the best evidence for the answer to the previous question?
(A) Lines 56–64 (“No service . . . war”)
(B) Lines 68–70 (“Mrs. Farquhar . . . hands”)
(C) Lines 70–73 (“While she . . . front”)
(D) Lines 104–106 (“He thanked . . . away”)
10. From Farquhar’s exchange with the soldier (lines 84–102), it can most reasonably be inferred that Farquhar is going to
(A) sneak across the bridge to join the Confederate forces.
(B) attempt to burn down the bridge to halt the Yankee advance.
(C) remove the driftwood blocking the Confederates’ access to the bridge.
(D) undermine the pillars that support the railroad bridge.
11. The main purpose of the concluding sentence of the passage is to
(A) offer an excuse for Farquhar’s failure to destroy the bridge.
(B) provide context useful in understanding Farquhar’s emotional reactions.
(C) establish that Farquhar has been entrapped into taking an unwise action.
(D) contrast Farquhar’s patriotic behavior with the scout’s treachery.
Questions 12–21 are based on the following passage.
The following passage is taken from Franklin Delano Roosevelt’s Third Inaugural Address, made on January 20, 1941, nearly a year before the bombing of Pearl Harbor triggered America’s entry into the Second World War.
Line (5) |
A nation, like a person, has something deeper, something more permanent, something larger than the sum of all its parts. It is that something which matters most to its future—which calls forth the most sacred guarding of its present. |
It is a thing for which we find it difficult—even impossible—to hit upon a single, simple word.
(10) (15) |
And yet we all understand what it is—the spirit—the faith of America. It is the product of centuries. It was born in the multitudes of those who came from many lands—some of high degree, but mostly plain people, who sought here, early and late, to find freedom more freely. |
(20) | The democratic aspiration is no mere recent phase in human history. It is human history. It permeated the ancient life of early peoples. It blazed anew in the middle ages. It was written in the Magna Carta. |
(25) | In the Americas its impact has been irresistible. America has been the New World in all tongues, to all peoples, not because this continent was a new-found land, but because all those who came here believed they could create upon this continent a new life—a life that should be new in freedom. |
(30) | Its vitality was written into our own Mayflower Compact, into the Declaration of Independence, into the Constitution of the United States, into the Gettysburg Address. |
(35) | Those who first came here to carry out the longings of their spirit, and the millions who followed, and the stock that sprang from them—all have moved forward constantly and consistently toward an ideal which in itself has gained stature and clarity with each generation. |
(40) | The hopes of the Republic cannot forever tolerate either undeserved poverty or self-serving wealth. |
(45) | We know that we still have far to go; that we must more greatly build the security and the opportunity and the knowledge of every citizen, in the measure justified by the resources and the capacity of the land. |
(50) | But it is not enough to achieve these purposes alone. It is not enough to clothe and feed the body of this Nation, and instruct and inform its mind. For there is also the spirit. And of the three, the greatest is the spirit. |
(55) | Without the body and the mind, as all men know, the Nation could not live. |
But if the spirit of America were killed, even though the Nation’s body and mind, constricted in an alien world, lived on, the America we know would have perished.
(60) (65) (70) |
That spirit—that faith—speaks to us in our daily lives in ways often unnoticed, because they seem so obvious. It speaks to us here in the Capital of the Nation. It speaks to us through the processes of governing in the sovereignties of 48 States. It speaks to us in our counties, in our cities, in our towns, and in our villages. It speaks to us from the other nations of the hemisphere, and from those across the seas—the enslaved, as well as the free. Sometimes we fail to hear or heed these voices of freedom because to us the privilege of our freedom is such an old, old story. |
(75) | The destiny of America was proclaimed in words of prophecy spoken by our first President in his first inaugural in 1789—words almost directed, it would seem, to this year of 1941: “The preservation of the sacred fire of liberty and the destiny of the republican model of government are justly (80) considered . . . deeply, . . . . finally, staked on the experiment intrusted to the hands of the American people.” |
(85) (90) |
If we lose that sacred fire—if we let it be smothered with doubt and fear—then we shall reject the destiny which Washington strove so valiantly and so triumphantly to establish. The preservation of the spirit and faith of the Nation does, and will, furnish the highest justification for every sacrifice that we may make in the cause of national defense. |
In the face of great perils never before encountered, our strong purpose is to protect and to perpetuate the integrity of democracy.
(95) | For this we muster the spirit of America, and the faith of America. |
We do not retreat. We are not content to stand still. As Americans, we go forward, in the service of our country, by the will of God.
12. As used in line 14, “plain” most nearly means
(A) candid.
(B) ordinary.
(C) homely.
(D) intelligible.
13. The author indicates which of the following about the American belief in freedom?
(A) It lacked any supporters who belonged to the upper classes.
(B) It had its origins at the time of the American Revolution.
(C) It is an ideal that has lost its hold on the public.
(D) It has deep-seated historical roots.
14. Which choice provides the best evidence for the answer to the previous question?
(A) Lines 1–3 (“A nation . . . parts”)
(B) Lines 17–21 (“The democratic . . . Carta”)
(C) Lines 23–28 (“America has been . . . freedom”)
(D) Lines 33–39 (“Those who first . . . generation”)
15. The author uses the Mayflower Compact, Declaration of Independence, Constitution, and Gettysburg Address as examples of
(A) subjects of previous inaugural addresses.
(B) expressions of the democratic aspiration.
(C) documents of historical interest.
(D) writings with ongoing legal implications.
16. The author recognizes counterarguments to the position he takes in lines 33–39 (“Those who first . . . generation”) by
(A) acknowledging that economic injustices must be addressed before democracy can prevail.
(B) admitting that the native-born descendents of our immigrant forebears have lost faith in democracy.
(C) conceding the lack of resources and capacity that hinder the fulfillment of the American dream.
(D) likening the Nation to a human body with physical, mental, and spiritual needs.
17. As used in line 76, “directed” most nearly means
(A) addressed.
(B) ordered.
(C) supervised.
(D) guided.
18. What main effect does the repetition of the phrase “It speaks to us” in lines 62–70 have on the tone of the passage?
(A) It creates a whimsical tone, endowing an abstract quality with a physical voice.
(B) It creates a colloquial tone, describing commonplace activities in ordinary words.
(C) It creates a dramatic tone, emphasizing the point being made and adding to its emotional impact.
(D) It creates a menacing tone, reminding us of our failure to heed the voices of freedom crying for our aid.
19. It can most reasonably be inferred that the experiment to which Washington refers in line 81 is
(A) a scientific investigation.
(B) a presidential inauguration.
(C) democratic government.
(D) national defense.
20. Which choice provides the best evidence for the answer to the previous question?
(A) Lines 56–59 (“But . . . perished”)
(B) Lines 60–62 (“That spirit . . . obvious”)
(C) Lines 83–87 (“If we . . . establish”)
(D) Lines 92–94 (“In the face . . . democracy”)
21. It is reasonable to conclude that a major goal of Roosevelt in making this speech was to
(A) inform American citizens of changes of policy in the new administration.
(B) impress his European counterparts with the soundness of America’s foreign policy.
(C) encourage American voters to avoid the divisiveness inherent in partisan politics.
(D) inspire the American people to defend the cause of freedom in dangerous times.
Questions 22–31 are based on the following passage.
This passage is from Mortal Lessons: Notes on the Art of Surgery, a classic book written by a contemporary American surgeon about his art.
Line (5) (10) (15) |
One holds the knife as one holds the bow of a cello or a tulip—by the stem. Not palmed nor gripped nor grasped, but lightly, with the tips of the fingers. The knife is not for pressing. It is for drawing across the field of skin. Like a slender fish, it waits, at the ready, then, go! It darts, followed by a fine wake of red. The flesh parts, falling away to yellow globules of fat. Even now, after so many times, I still marvel at its power—cold, gleaming, silent. More, I am still struck with dread that it is I in whose hand the blade travels, that my hand is its vehicle, that yet again this terrible steel-bellied thing and I have conspired for a most unnatural purpose, the laying open of the body of a human being. |
(20) (25) |
A stillness settles in my heart and is carried to my hand. It is the quietude of resolve layered over fear. And it is this resolve that lowers us, my knife and me, deeper and deeper into the person beneath. It is an entry into the body that is nothing like a caress; still, it is among the gentlest of acts. Then stroke and stroke again, and we are joined by other instruments, hemostats and forceps, until the wound blooms with strange flowers whose looped handles fall to the sides in steely array. |
(30) (35) (40) |
There is a sound, the tight click of clamps fixing teeth into severed blood vessels, the snuffle and gargle of the suction machine clearing the field of blood for the next stroke, the litany of monosyllables with which one prays his way down and in: clamp, sponge, suture, tie, cut. And there is color. The green of the cloth, the white of the sponges, the red and yellow of the body. Beneath the fat lies the fascia, the tough fibrous sheet encasing the muscles. It must be sliced and the red beef of the muscles separated. Now there are retractors to hold apart the wound. Hands move together, part, weave. We are fully engaged, like children absorbed in a game or the craftsmen of some place like Damascus. |
(45) (50) (55) (60) |
Deeper still. The peritoneum, pink and gleaming and membranous, bulges into the wound. It is grasped with forceps, and opened. For the first time we can see into the cavity of the abdomen. Such a primitive place. One expects to find drawings of buffalo on the walls. The sense of trespassing is keener now, heightened by the world’s light illuminating the organs, their secret colors revealed—maroon and salmon and yellow. The vista is sweetly vulnerable at this moment, a kind of welcoming. An arc of the liver shines high and on the right, like a dark sun. It laps over the pink sweep of the stomach, from whose lower border the gauzy omentum is draped, and through which veil one sees, sinuous, slow as just-fed snakes, the indolent coils of the intestine. |
(65) | You turn aside to wash your gloves. It is a ritual cleansing. One enters this temple doubly washed. Here is man as microcosm, representing in all his parts the Earth, perhaps the universe. |
(70) (75) |
I must confess that the priestliness of my profession has ever been impressed on me. In the beginning there are vows, taken with all solemnity. Then there is the endless harsh novitiate of training, much fatigue, much sacrifice. At last one emerges as a celebrant, standing close to the truth lying curtained in the ark of the body. Not surplice and cassock but mask and gown are your regalia. You hold no chalice, but a knife. There is no wine, no wafer. There are only the facts of blood and flesh. |
22. The passage is best described as
(A) a definition of a concept.
(B) an example of a particular method.
(C) a lesson on a technique.
(D) a description of a process.
23. It can most reasonably be inferred from the passage that the “wake of red” to which the author refers (line 7) is
(A) a sign of embarrassment.
(B) an infectious rash.
(C) a line of blood.
(D) the blade of the knife.
24. Which choice provides the best evidence for the answer to the previous question?
(A) Lines 1–2 (“One . . . stem”)
(B) Lines 2–4 (“Not . . . fingers”)
(C) Lines 7–8 (“The flesh . . . fat”)
(D) Lines 17–18 (“A stillness . . . hand”)
25. As used in line 8, “parts” most nearly means
(A) leaves.
(B) splits.
(C) surrenders.
(D) distributes.
26. As used in line 42, “engaged” most nearly means
(A) betrothed.
(B) engrossed.
(C) hired.
(D) embattled.
27. In lines 49–50, the comment “One expects to find drawings of buffalo on the walls” metaphorically compares the abdominal cavity to
(A) an art gallery.
(B) a zoological display.
(C) a Western film.
(D) a prehistoric cave.
28. The author most likely describes the colors of the internal organs as “secret” (line 52) because
(A) they are beyond ordinary human understanding.
(B) they normally are hidden from sight.
(C) their access is limited to authorized personnel.
(D) they are darker in color than the external organs are.
29. In creating an impression of abdominal surgery for the reader, the author primarily makes use of
(A) comparison with imaginary landscapes.
(B) contrast to other types of surgery.
(C) references to religious imagery.
(D) evocation of the patient’s emotions.
30. Which choice provides the best evidence for the answer to the previous question?
(A) Lines 36–40 (“Beneath the fat . . . wound”)
(B) Lines 44–48 (“The peritoneum . . . abdomen”)
(C) Lines 54–57 (“The vista . . . sun”)
(D) Lines 62–64 (“It is a . . . washed”)
31. One aspect of the passage that may make it difficult to appreciate is the author’s apparent assumption throughout that readers will
(A) have qualms about reading descriptions of major surgery.
(B) be already adept at handling surgical tools.
(C) be familiar with the organs and tissues that are named.
(D) relate accounts of specific surgical acts to their own experience of undergoing surgery.
Questions 32–42 are based on the following passages.
Passage 1 is taken from a historical study, done in the 1980s, of the relationship between the press and each American president from George Washington to Ronald Reagan. Passage 2 is taken from a 2006 master’s thesis on the relationship between the president and the press during the first term of President George W. Bush.
PASSAGE 1
Line (5) (10) |
In the shifting relationship between the press and the presidency over nearly two centuries, there has remained one primary constant—the dissatisfaction of one with the other. No president has escaped press criticism, and no president has considered himself fairly treated. The record of every administration has been the same, beginning with mutual protestations of goodwill, ending with recriminations and mistrust. |
(15) (20) (25) |
This is the best proof we could have that the American concept of a free press in a free society is a viable idea, whatever defects the media may have. While the Founding Fathers and their constituencies did not always agree on the role the press should play, there was a basic consensus that the newspaper (the only medium of consequence at the time) should be the buffer state between the rulers and the ruled. The press could be expected to behave like a watchdog, and government at every level, dependent for its existence on the opinions of those it governed, could expect to resent being watched and having its shortcomings, real or imaginary, exposed to the public view. |
(30) (35) |
Reduced to such simple terms, the relationship of the presidents to the press since George Washington’s first term is understandable only as an underlying principle. But this basic concept has been increasingly complicated by the changing nature of the presidency, by the individual nature of presidents, by the rise of other media, especially television, and by the growing complexity of beliefs about the function of both press and government. |
(40) (45) (50) |
In surveying nearly two centuries of this relationship, it is wise to keep in mind an axiom of professional historians—that we should be careful not to view the past in terms of our own times, and make judgments accordingly. Certain parallels often become obvious, to be sure, but to assert what an individual president should or should not have done, by present standards, is to violate historical context. Historians occasionally castigate each other for this failing, and in the case of press and government, the danger becomes particularly great because the words themselves—”press” and “government,” even “presidency”—have changed in meaning so much during the past two hundred years. |
(55) (60) (65) (70) |
It is part of American mythology that the nation was “cradled in liberty” and that the colonists, seeking religious freedom, immediately established a free society, but the facts are quite different. The danger of an uncontrolled press to those in power was well expressed by Sir William Berkeley, governor of Virginia, when he wrote home to his superiors in 1671: “I thank God there are no free schools nor printing, and I hope we shall not have these hundred years; for learning has brought disobedience, and heresy, and sects into the world, and printing has divulged them, and libels against the best government, God keep us from both.” There are those in twentieth-century America who would say “Amen” to Berkeley’s view of printing and “libels against the best government.” |
PASSAGE 2
(75) (80) (85) (90) (95) |
In their analysis of aggressive journalist behavior in a comparative study of press conferences held by Presidents Eisenhower and Reagan, Clayman and Heritage (2002) developed an original encoding system according to ten different features of question design. Their findings showed significantly greater levels of aggression and adversarial behavior by the press in dealings with the more recent president. Clayman, Elliot, Heritage & McDonald’s updated study (2004) refined the coding process and used a more continuous sample to test the validity and reliability of the original study. Their comparison of journalistic adversarialness covered each president from Eisenhower to Clinton and supported original results that show a long-term decline in deference to the president. The continuous sample revealed more volatility than the simpler work on which it was based but is a further testament to the increased aggressiveness, sometimes adversarial treatment prevalent in press conferences regardless of partisanship or personal idiosyncrasy. |
(100) (105) (110) |
These findings would suggest that the increasingly contentious, adversarial relationship between the press and the highest ranking executive official has created a modern press conference where the president must relinquish more agenda-setting control than in other communicative processes. In each session, he subjects himself to open questioning that is shown to be significantly less deferential, more direct and often more aggressive and hostile than ever before. This would seem an appropriate justification for the dwindling numbers of traditional solo press conferences in recent administrations (Kumar, 2003b). |
Table 1
Solo and Joint Press Conferences by President 1981–2004
President | Total | Solo | Joint | Joint Sessions as Percent of Total |
Reagan* | 46 | 46 | 0 | 00.0% |
George H. W. Bush* | 142 | 83 | 59 | 41.5% |
Clinton* | 193 | 62 | 131 | 67.9% |
George W. Bush** | 88 | 88 | 68 | 77.3% |
*Cited in Kumar, 2003b
**Compiled from Weekly Compilation of Presidential Documents
32. The main purpose of Passage 1 is to
(A) examine methods of evaluating the relationship between the press and the president.
(B) argue that the adversarial relationship between the press and the presidency has proven deleterious to both.
(C) present an overview of an inherently conflicted relationship that faces new challenges.
(D) consider a political dilemma created by the mutual antagonism between two major institutions.
33. According to the opening paragraph of Passage 1, all American presidents have experienced
(A) defects in the quality of their press coverage.
(B) goodwill from some reporters in the press corps.
(C) alternating periods of antagonism and harmony with the press.
(D) mutual animosity involving themselves and the press.
34. Which choice provides the best evidence for the answer to the previous question?
(A) Lines 7–10 (“The record . . . mistrust”)
(B) Lines 11–14 (“This . . . may have”)
(C) Lines 14–20 (“While . . . ruled”)
(D) Lines 27–31 (“Reduced . . . principle”)
35. As used in line 27, “reduced” most nearly means
(A) decreased.
(B) boiled down.
(C) marked down.
(D) demoted.
36. The authors of Passage 1 caution the reader about judging the actions of long-dead presidents because
(A) historical accounts, when investigated, have proven to be untrustworthy.
(B) contemporary authors have rewritten history to reflect current academic opinions.
(C) readers today cannot fully grasp the significance these actions had in their own time.
(D) history, at best, is an imprecise science.
37. Which choice provides the best evidence for the answer to the previous question?
(A) Lines 1–7 (“In the shifting . . . treated”)
(B) Lines 20–26 (“The press . . . public view”)
(C) Lines 27–37 (“Reduced . . . government”)
(D) Lines 43–54 (“Certain parallels . . . years”)
38. In the opening sentence of the final paragraph (lines 55–59) of Passage 1, the authors seek primarily to
(A) define a term.
(B) defend a widely held belief.
(C) correct a misconception.
(D) champion a cause.
39. As used in line 107, “open” most nearly means
(A) receptive.
(B) unrestricted.
(C) unconcealed.
(D) vulnerable.
40. Data in the graph about presidential solo and joint press conferences from 1981–2004 most strongly support which of the following statements?
(A) President Clinton held more solo press conferences than President George H. W. Bush did.
(B) Presidents Clinton and George W. Bush held a far higher percentage of joint press conferences than either of their predecessors did.
(C) President Reagan’s failure to hold joint press conferences resulted from a reluctance to share the spotlight with other members of his administration.
(D) While President George H. W. Bush held far more press conferences than his son President George W. Bush did, both Presidents Bush held more joint sessions than solo sessions.
41. Which choice best describes the relationship between the two passages?
(A) Passage 2 denies the static nature of the phenomenon described in Passage 1.
(B) Passage 2 evaluates the conclusions drawn from assertions made in Passage 1.
(C) Passage 2 predicts the eventual healing of a breach reported in Passage 1.
(D) Passage 2 critiques the hypotheses proposed by researchers cited in Passage 1.
42. On which of the following points would the authors of both passages most likely agree?
(A) Those who criticize the press for its treatment of the president fail to understand the press’s watchdog function.
(B) Members of the press corps are unlikely to prefer joint press conferences to solo sessions.
(C) The relationship between the press and the presidency is inherently adversarial, and likely to remain so.
(D) The president needs to regain agenda-setting control of traditional solo press conferences.
Questions 43–52 are based on the following passage.
The following passage is abridged from Rachel Ehrenberg’s “The facts behind the frack” (Science News), an article on the controversies surrounding the hydraulic fracturing method of recovering natural gas from below the Earth’s surface.
To call it a fractious debate is an understatement.
Line (5) (10) |
Hydraulic fracturing, or fracking, wrenches open rock deep beneath the Earth’s surface, freeing the natural gas that’s trapped inside. Proponents argue that fracking-related gas recovery is a game changer, a bridge to the renewable energy landscape of the future. The gas, primarily methane, is cheap and relatively clean. Because America is brimful of the stuff, harvesting the fuel via fracking could provide the country with jobs and reduce its dependence on foreign sources of energy. |
(15) (20) |
But along with these promises have come alarming local incidents and national reports of blowouts, contamination and earthquakes. Fracking opponents contend that the process poisons air and drinking water and may make people sick. What’s more, they argue, fracking leaks methane, a potent greenhouse gas that can blow up homes, worries highlighted in the controversial 2010 documentary Gasland. |
(25) (30) (35) |
Fears that fracking companies are operating in a Wild West environment with little regulation have prompted political action. In June, the group Don’t Frack Ohio led thousands of protesters on a march to the statehouse, where they declared their commitment to halting hydraulic fracturing in the state. Legislation banning the process has been considered but is now on hold in California. New York—which sits atop a giant natural gas reserve—has a statewide fracking moratorium; pending policies would allow the process only where local officials support it. |
(40) (45) (50) (55) |
Despite all this activity, not much of the fracking debate has brought scientific evidence into the fold. Yet scientists have been studying the risks posed by fracking operations. Research suggests methane leaks do happen. The millions of gallons of chemical-laden water used to fracture shale deep in the ground has spoiled land and waterways. There’s also evidence linking natural gas recovery to earthquakes, but this problem seems to stem primarily from wastewater disposal rather than the fracturing process itself. While the dangers are real, most problems linked to fracking so far are not specific to the technology but come with many large-scale energy operations employing poor practices with little oversight, scientists contend. Whether the energy payoff can come with an acceptable level of risk remains an open question. |
(60) (65) (70) (75) |
Hydraulic fracturing operations have been linked to some small earthquakes, including a magnitude 2.3 quake near Blackpool, England, last year. But scientists agree such earthquakes are extremely rare, occurring when a well hits a seismic sweet spot, and are avoidable with monitoring. Of greater concern are earthquakes associated with the disposal of fracking fluid into wastewater wells. Injected fluid essentially greases the fault, a long-known effect. In the 1960s, a series of Denver earthquakes were linked to wastewater disposal at the Rocky Mountain arsenal, an Army site nearby. Wastewater disposal was also blamed for a magnitude 4.0 quake in Youngstown, Ohio, last New Year’s Eve. |
(80) (85) (90) |
A study headed by William Ellsworth of the U.S. Geological Survey in Menlo Park, Calif., documents a dramatic increase in earthquakes in the Midwest coinciding with the start of the fracking boom. From 1970 to 2000, the region experienced about 20 quakes per year measuring at or above magnitude 3.0. Between 2001 and 2008, there were 29 such quakes per year. Then there were 50 in 2009, 87 in 2010 and 134 in 2011. “The change was really quite pronounced,” says Ellsworth. “We do not think it’s a purely natural phenomenon.” However, the earthquakes weren’t happening near active drilling—they seemed to be clustered around wastewater wells. |
(95) (100) (105) (110) |
It’s hard to look back without pre-quake data and figure out what triggers a single earthquake, notes Ellsworth. There are several pieces of the geology equation that, if toggled, can tip a fault from stable to unstable. A recent study examining seismic activity at wastewater injection wells in Texas linked earthquakes with injections of more than 150,000 barrels of water per month. But not every case fits the pattern, suggesting the orientation of deep faults is important. Ellsworth advises that injection at active faults be avoided. Drill sites should be considered for their geological stability, and seismic information should be collected. (Only about 3 percent of the 75,000-odd hydraulic fracturing setups in the United States in 2009 were seismically monitored.) “There are many things we don’t understand,” says Ellsworth. “We’re in ambulance-chasing mode where we’re coming in after the fact.” |
After decades of a steady earthquake rate (dotted line) in the central and eastern United States, activity began to rise in about 2009 and jumped to five times the normal rate by 2013, probably due to human activity.
Source: W. L. Ellsworth/Science 2013
43. In line 1, the author chooses the word “fractious” (contentious; heated) to create
(A) a metaphor.
(B) a play on words.
(C) an exaggeration.
(D) a counterargument.
44. To call fracking-related gas recovery “a game changer” (line 7) is to assert that fracking
(A) has no foreseeable negative consequences.
(B) will radically alter natural gas production.
(C) is not taken seriously by its proponents.
(D) will require active federal regulation.
45. Which choice provides the best evidence for the answer to the previous question?
(A) Lines 9–14 (“The gas . . . energy”)
(B) Lines 15–20 (“But . . . sick”)
(C) Lines 20–24 (“What’s more . . . Gasland”)
(D) Lines 51–59 (“While . . . question”)
46. What function does the discussion of fracking legislation in lines 32–38 serve in the passage?
(A) It describes specific responses to concerns raised in the previous paragraph.
(B) It analyzes theoretical objections to a claim made in the previous paragraph.
(C) It provides an unanticipated reaction to an explicit demand made in the previous paragraph.
(D) It contradicts a working hypothesis proposed in the previous paragraph.
47. As used in line 58, “open” most nearly means
(A) unresolved.
(B) vulnerable.
(C) accessible.
(D) ajar.
48. The stance that the author takes throughout the passage is best described as that of
(A) an advocate of technological innovations.
(B) an opponent of pointless regulatory oversight.
(C) a legislator concerned about potential danger.
(D) an observer striving to present a balanced account.
49. Which choice provides the best evidence for the answer to the previous question?
(A) Lines 25–28 (“Fears . . . political action”)
(B) Lines 32–38 (“Legislation . . . support it”)
(C) Lines 51–59 (“While . . . question”)
(D) Lines 93–95 (“It’s hard . . . Ellsworth”)
50. The graph based on Ellsworth’s figures accentuates the
(A) validity of his research team’s methodology.
(B) increased magnitude of each individual earthquake.
(C) increasing frequency of earthquakes in the region.
(D) amount of fracking fluid injected into wastewater wells.
51. As used in line 87, “pronounced” most nearly means
(A) noticeable.
(B) declared.
(C) decided on.
(D) articulated.
52. It can be most reasonably inferred from the concluding paragraph that Ellsworth looks on current hypotheses about connections between the recent increases in earthquakes and the start of the fracking boom as
(A) corroborated by pre-quake data.
(B) evidence of seismic activity.
(C) based on insufficient knowledge.
(D) contradicted by his research findings.
If there is still time remaining, you may review your answers.
WRITING AND LANGUAGE TEST
35 MINUTES, 44 QUESTIONS
Turn to Section 2 of your answer sheet to answer the questions in this section.
Directions: Questions follow each of the passages below. Some questions ask you how the passage might be changed to improve the expression of ideas. Other questions ask you how the passage might be altered to correct errors in grammar, usage, and punctuation. One or more graphics accompany some passages. You will be required to consider these graphics as you answer questions about editing the passage.
There are three types of questions. In the first type, a part of the passage is underlined. The second type is based on a certain part of the passage. The third type is based on the entire passage.
Read each passage. Then, choose the answer to each question that changes the passage so that it is consistent with the conventions of standard written English. One of the answer choices for many questions is “NO CHANGE.” Choosing this answer means that you believe the best answer is to make no change in the passage.
Questions 1–11 are based on the following passage.
Modernism can be characterized by its complete rejection of 19th-century traditions and values of prudish and proper etiquette. F. Scott Fitzgerald’s “Bernice Bobs Her Hair” was written in 1920 and reflects this embrace of conventional morality most effectively through the character of Marjorie Harvey. Marjorie, an immensely popular and desirable young woman, is plagued by Bernice, her dull cousin who fails to entertain
or be entertained by Marjorie’s many social environments. In a desperate attempt to make Bernice more popular and therefore, more bearable, Marjorie teaches Bernice to appear beautifully at ease with
itself in order to gain social favor. Fitzgerald uses Bernice’s transformation to embody Modernist ideals of moral relativism and
the implementation of mockery of former Victorian standards of custom.
Marjorie, a quintessential modern girl, represents the destruction of conventional norms and former ideas of femininity. Young and beautiful, she is interested only in having a good time and being good company to the many suitors whom flock to her. Despite her good looks and family wealth, Bernice is disliked for her stifling and overly formal Victorian propriety.
On the other hand, Bernice is old-fashioned, outdated, and unpopular.
The “new,” modern woman is best denoted by her wit, carelessness, and lack of emotion. Where the dignified nature of Bernice is seen as snobbish and out of style, Marjorie’s sardonic and indifferent manner is fresh and exciting. The stark contrast between the Victorian and Modernist eras is even depicted in the girls’ taste in literature: Marjorie casts off Bernice’s reference to Little Women in exchange for the more recent Oscar Wilde.
Still, Modernism isn’t let off easy in Fitzgerald’s well-liked short story. When Marjorie is preferred socially, she is flagrantly rude and always needing to be entertained. She instructs Bernice in social protocol in a
few short sentences, causing the reader to question the frivolous hedonism that dominates the early 20th century. Once Bernice adopts her cousin’s apathy, she easily falls into the world of dancing, dating, and laughing. In fact, never being serious happens to come quite easy.
The equally kind-hearted natures of both of Fitzgerald’s characters come crashing down when Marjorie tricks Bernice into getting her hair bobbed—a style so rebellious that it causes Bernice to faint. Bernice finds revenge in severing off a golden lock of Marjorie’s hair while she sleeps. While using Bernice and Marjorie to model both eras, Fitzgerald finds flaws in
both: the old manner is a lifeless forgery, while the new approach is only relaxed on the surface.
1. Which wording is most consistent with the paragraph as a whole?
(A) NO CHANGE
(B) ignorance
(C) rebuff
(D) significance
2. (A) NO CHANGE
(B) and entertainment
(C) with the entertaining of
(D) of the entertaining for
3. (A)NO CHANGE
(B) oneself
(C) themselves
(D) herself
4. (A) NO CHANGE
(B) for the mocking of
(C) to mock
(D) mocking
5. (A) NO CHANGE
(B) who
(C) whose
(D) who’s
6. Where in this paragraph should the underlined sentence be placed?
(A) where it is now
(B) before the first sentence
(C) before the second sentence
(D) before the third sentence
7. (A) NO CHANGE
(B) among
(C) for
(D) on
8. (A) NO CHANGE
(B) While
(C) Because
(D) Since
9. (A) NO CHANGE
(B) short few
(C) few, short
(D) short, few
10. Which choice would best be logically placed here to represent the characterizations of Marjorie and Bernice in the passage?
(A) NO CHANGE
(B) revolutionary dogmatism
(C) false facades
(D) frivolous piety
11. (A) NO CHANGE
(B) both, the old manner is a lifeless forgery while
(C) both—the old manner is a lifeless, forgery, while
(D) both; the old manner, is a lifeless forgery while
Questions 12–22 are based on the following passage and supplementary material.
If any field has drastically changed in the last two decades, it is journalism. Journalism includes the gathering and distribution of news through a variety of mediums, building upon the long-standing professional excellence with which journalism is associated. Whether via print, broadcast, or digital, journalists are responsible for keeping the public informed, and often play a vital role in allowing the general population to participate in the political process. Although the digital age has understandably discouraged popularity in some traditional forms of
news media the field itself is optimistic, not only is the digital platform more than making up for the moderate declines in traditional news sources,
but also research shows that Americans are spending more time consuming news than they have since the early 1990s.
The traditional dominance of newspapers has continued unabated.
Quite simply, the days of print-only newsrooms are past. Now, one doesn’t wait until the 6 P.M. broadcast to hear what’s happening around the world, nor does one grab the newspaper on Sunday morning for breaking news. The public expects minute-by-minute updates, and media companies meet this demand with 24-7 online newsreels. Journalists can no longer limit themselves to gathering stories or writing articles or speaking publicly—they must be able to do it all and then some. Even entry-level positions require candidates who have had media training and internship experience in addition to a formal education. Internships at most media outlets include everything from copy editing to blogging.
The tough competition and demanding prerequisites for the job market need not be deterrents. Leading journalism department’s are reassuring that their students leave undergraduate with all the tools necessary for success. For instance, the University of Missouri at Columbia
—boasting the number one journalism department in the nation according to The Huffington Post—offers more than 30 interest areas, incorporating an intensive liberal arts education along with hands-on experience in media labs and internships for academic credit. Ohio
University also having, a journalism department ranked in the top ten nationwide offers three campus publications plus a broadcasting outlet for students to gain professional experience before graduation, not to mention OU’s Institute for International Journalism, which offers opportunities for reporting abroad.
Technology and its endless affects on all areas of the job market are tedious subjects for the student and young professional. One cannot consider a career field without hearing how formidable its outlook is and how quickly one could fail in an uncertain economy. Indeed, journalism students have been well informed
about the steadily increasing demand for journalists in the recent past, but the truth stands that there will always be a demand for the news, and therefore, a need for journalists. The field
is adapting and so are its constituents.
12. Which choice most specifically elaborates on the first part of this sentence?
(A) NO CHANGE
(B) growing its reach to include urban, suburban, and rural population centers.
(C) which have recently expanded to incorporate smartphones, tablets, and blogs.
(D) demonstrating that seeking the average public opinion is most objective.
13. (A) NO CHANGE
(B) news media, the field itself is optimistic, not only
(C) news media, the field itself is optimistic: not only
(D) news media the field itself; is optimistic not only
14. (A) NO CHANGE
(B) and
(C) for
(D) since
15. Which choice best concludes this paragraph and transitions to the topic of the next paragraph?
(A) NO CHANGE
(B) Journalism isn’t dying; the way reporters do their job is changing.
(C) Journalism is no longer the sort of career that globally minded people would chose.
(D) With the steady demise of public interest in quality journalism, it is only a matter of time before journalism falls by the wayside.
16. (A) NO CHANGE
(B) because
(C) for
(D) while
17. (A) NO CHANGE
(B) departments’ are insuring that they’re
(C) departments are assuring there
(D) departments are ensuring that their
18. Which choice best connects this sentence to the previous sentence?
(A) NO CHANGE
(B) —located in the geographic near-middle of the United States—
(C) —a university that offers a variety of possible undergraduate majors and minors—
(D) —ranked among the best universities for average starting salary among its graduates—
19. (A) NO CHANGE
(B) University also having a journalism department ranked in the top ten nationwide offers
(C) University, also having a journalism department, ranked in the top ten, nationwide, offers
(D) University, also having a journalism department ranked in the top ten nationwide, offers
20. (A) NO CHANGE
(B) endless effects
(C) endlessly affects
(D) endlessly effects
21. Which choice offers the most accurate interpretation of the data in the chart?
(A) NO CHANGE
(B) about the gradual decline in jobs for journalists in the past decade,
(C) about the constant level of employment for journalists these past few years,
(D) about the job market fluctuations in recent years,
22. (A) NO CHANGE
(B) was adapting
(C) is adopting
(D) was adopting
Questions 23–33 are based on the following passage.
Of all the ancient, sacred, and truly splendid buildings to visit, the Parthenon may just be the most treasured of all. A long time past, the Greeks built their apotheosis over a span of nine years atop the Acropolis of Athens as a tribute to Athena, the city’s beloved patron goddess of war and reason. The temple itself was completed in 438 B.C., although decorative sculpting and engraving within the structure went on for several more years. Since then, the structure has served as
temple, treasury, church, and most recently, tourist attraction.
Pericles—leading politician in 5th century B.C.—recruited the sculptor Phidias to oversee two architects, Iktinos and Kallikrates, in the construction of the Parthenon to house a forty-foot high statue of Athena. Honestly and judiciously, the ancient Greeks planned an exceptional monument with a base the size of half a football field and pillars over thirty feet tall. Athenians stored their most lavish possessions inside the Parthenon among a host of statues, sculptures, precious metals, and treasures taken in the conquest of the Persians.
Yet, the endeavor and all it stood for were short-lived: just seven years after the Parthenon was constructed, war broke out with Sparta. Sometime after the reign of Athens, in 5th century A.D., the statue of Athena was plundered and later destroyed.
Perhaps, even with Athena—the very core of Parthenon—missing, the temple could of still served as a great, inclusive museum of Greek history, tracing the founding of Ancient Greece, Athenian democracy, and early western civilization; yet, the Parthenon would endure many other foes. The Parthenon was first converted to a Christian church, which led to the removal of
its’ “pagan gods.” With the rise of the Ottoman Empire, the monument was used as a mosque until a Venetian attack on Athens destroyed large parts of the building and left its
archaeology deserted. By the 18th century, little was left of the Parthenon after decades of European pillaging.
In the contemporary world in which we reside, the Parthenon is one of the most popular tourist attractions in the world, enticing millions of people each year and warranting an ongoing restoration project currently in its third decade. Even in its antiquity, its subtle beauty and architectural refinement
is uncontested. Its miracle comes not from its magnitude, but from the curvatures between its platform and columns that offer an illusion of symmetry that exceeds its true dimensions, and in the elaborate engravings within its marble surfaces
that having to outlast centuries of calamity. Now, architects, engineers, and artists work to recreate the surprisingly balanced and unbelievably precise work of the Athenians.
How is it that today’s architects are taking forty years to do what they did in less than ten?
23. Which choice would most specifically describe how long ago the Parthenon was constructed?
(A) NO CHANGE
(B) More than 2,500 years ago,
(C) Many decades of ages past,
(D) In days gone by,
24. (A) NO CHANGE
(B) temple, treasury church, and most recently, tourist attraction.
(C) temple treasury, church and most recently tourist attraction.
(D) temple treasury church, and most recently tourist attraction.
25. What could best be used for the underlined portion to convey the high priority the Greeks placed on completing the Parthenon in an extravagant fashion?
(A) NO CHANGE
(B) Sparing no expense,
(C) With artistic patience,
(D) Using architectural techniques,
26. (A) NO CHANGE
(B) Additionally,
(C) In conclusion,
(D) As a result,
27. (A) NO CHANGE
(B) might of
(C) could have
(D) should have been
28. (A) NO CHANGE
(B) it’s
(C) it is
(D) its
29. (A) NO CHANGE
(B) components
(C) particles
(D) remnants
30. (A) NO CHANGE
(B) In the world of today,
(C) Contemptuously,
(D) Today,
31. (A) NO CHANGE
(B) are
(C) was
(D) were
32. (A) NO CHANGE
(B) which has to outlast
(C) that have outlasted
(D) which had outlasted
33. Which of the following would be the most effective conclusion to the essay?
(A) NO CHANGE
(B) It is vital that we learn from the past in order to not repeat the mistakes of history.
(C) Tourism is a growing business worldwide, as people seek out memorable experiences rather than to accumulate possessions.
(D) The world continues to be haunted by the Venetian attack on the Parthenon, turning a brilliant accomplishment into utter ruins.
Questions 34–44 are based on the following passage.
All humans have their ultimate genetic roots in Africa. While our own ancestors were battling drought on the coasts of the African sub-continent,
the icebound north of modern Eurasia experienced the spread of the evolutionarily distinct species Homo neanderthalensis, where the Neanderthals developed the tools of flint and bone that have today come to characterize the so-called Mousterian culture of the early Stone Age.
(1) Early hypotheses for their extinction centered, predictably, around the climate extreme change of the last Ice Age. (2) However, more recent studies of Neanderthal anatomy and artifacts suggest that they were remarkably well-equipped to deal with the fiercely cold and barren conditions,
and even thrived within them for nearly 200,000 years. (3) To cope with the glacial conditions, Neanderthals became short in stature—no more than a meter and half tall—and developed short, broad extremities that would have increased the efficiency of circulation, and helped to preserve body heat.
Another popular theory posits that Neanderthals met their extinction through absorption. That is—supposing Neanderthals were not a distinct species, but rather a subspecies of Homo sapiens—some researchers believe that they disappeared after conflicts with humans when they arrived in Eurasia roughly 80,000 years ago. However, a sample of mitochondrial DNA surviving in the remains of a Neanderthal discovered in the Caucus Mountains demonstrates 3.5 percent genetic divergence from
contemporary Homo sapiens. While it is possible that some Neanderthals may have become culturally assimilated with our ancestors, it is highly unlikely that their DNA contributed to that of modern humans.
Currently, the most widely held theory to explain the extinction of the Neanderthals boils down quite simply to the processes of natural selection. While Neanderthals appear to have maintained a stable population during the Ice Age, a drastic genetic bottleneck was experienced by our African ancestors, leaving only the strongest and most intelligent to survive and carry on the species. When Homo neanderthalensis at last met Homo sapiens, it is probable that
they was outmatched, at the very least, in technology, creativity, and social efficacy. In the several thousand years that followed, competition for resources would have pushed Neanderthals farther and farther to the
oceans of Europe and Asia. The last known remnants of Neanderthal culture issue from the remote location of Gorham’s Cave on the Gibraltar coast. By this time—roughly 27,000 years ago—Homo neanderthalensis had been displaced by its evolutionary cousin
to the very edge of the land nearly back into Africa itself where our common ancestors, first emerged millions of years prior.
34. Which choice would best function as the introductory thesis of the essay?
(A) NO CHANGE
(B) The defeat of the Neanderthal invaders can only be considered a triumph of human ingenuity.
(C) The disappearance of the Neanderthals is one of the great mysteries in the evolutionary success of modern humans.
(D) In order to cope with the repercussions of possible global climate change, we should look to the example of Neanderthal adaptation.
35. (A) NO CHANGE
(B) the evolutionarily distinct species Homo neanderthalensis had spread to the icebound north of modern Eurasia,
(C) the species Homo neanderthalensis, being evolutionarily distinct, found itself spread to modern Eurasia in the north icebound,
(D) the north icebound of modern Eurasia experience evolutionarily distinct species spread of the Homo neanderthalensis,
36. (A) NO CHANGE
(B) climate, extreme
(C) extreme climate
(D) extreme, climate
37. (A) NO CHANGE
(B) but
(C) for it was the case that they
(D) OMIT the underlined portion.
“Further, there is strong evidence to suggest that later Neanderthals were capable of creating sophisticated and versatile garments from animal pelts designed to maintain core warmth without inducing perspiration.”
The best placement for this sentence is
(A) before sentence 1
(B) before sentence 2
(C) before sentence 3
(D) after sentence 3
39. Which choice is the most consistent elaboration on the first sentence of this paragraph?
(A) NO CHANGE
(B) interbreeding
(C) discoveries
(D) commerce
40. Which wording best conveys that the Neanderthals only have a slight genetic divergence from present-day humans?
(A) NO CHANGE
(B) punctual
(C) unique
(D) scientific
41. (A) NO CHANGE
(B) a drastic genetic bottleneck by our African ancestors was experienced,
(C) our African ancestors drastically experienced a bottleneck that was genetic,
(D) our African ancestors experienced a drastic genetic bottleneck,
42. (A) NO CHANGE
(B) they were
(C) the Neanderthals are
(D) the Neanderthals were
43. (A) NO CHANGE
(B) margins
(C) debris
(D) remains
44. (A) NO CHANGE
(B) to the very edge, of the land nearly back into Africa itself, where our common ancestors
(C) to the very edge of the land, nearly back into Africa itself, where our common ancestors
(D) to the very edge of the land nearly, back into Africa itself where our common, ancestors
If there is still time remaining, you may review your answers.
MATH TEST (NO CALCULATOR)
25 MINUTES, 20 QUESTIONS
Turn to Section 3 of your answer sheet to answer the questions in this section.
Directions: For questions 1-15, solve each problem and choose the best answer from the given choices. Fill in the corresponding circle on your answer sheet. For questions 16-20, solve each problem and enter your answer in the grid on your answer sheet.
Notes:
Calculators are NOT PERMITTED in this section.
All variables and expressions represent real numbers unless indicated otherwise.
All figures are drawn to scale unless indicated otherwise.
All figures are in a plane unless indicated otherwise.
Unless indicated otherwise, the domain of a given function is the set of all real numbers x for which the function has real values.
REFERENCE INFORMATION
The arc of a circle contains 360°.
The arc of a circle contains 2π radians.
The sum of the measures of the angles in a triangle is 180°.
1. At the beginning of January, John deposits A dollars into a non-interest-bearing bank account. If John withdraws d dollars from the account every month and makes no additional deposits, how much money, in dollars, will be in the account after m months?
(A) A – md
(B) (A – m)d
(C)
(D)
2. If f(x) = x2 – 11, for what values of x is f(x) < 25?
(A) –6 < x
(B) x < 6
(C) x ≤ –6 or x ≥ 6
(D) –6 < x < 6
3. At Joe’s Pizzeria, small pizzas cost $7.50 and large pizzas cost $11.00. One day from 3:00 P.M. to 9:00 P.M., Joe sold 100 pizzas and took in $848. Solving which of the following systems of equations could be used to determine the number of small pizzas, S, and the number of large pizzas, L, that Joe sold during that 6-hour period?
(A)
(B)
(C)
(D)
4. Which of the following statements is true concerning the equation below?
3(5 – 2x) = 6(2 – x) + 3
(A) The equation has no solutions.
(B) The equation has one positive solution.
(C) The equation has one negative solution.
(D) The equation has infinitely many solutions.
5. The chart below shows the value of an investment on January 1 of each year from 2005 to 2010. During which year was the percent increase in the value of the investment the greatest?
Year | Value |
2005 | $150 |
2006 | $250 |
2007 | $450 |
2008 | $750 |
2009 | $1,200 |
2010 | $1,800 |
(A) 2005
(B) 2006
(C) 2008
(D) 2009
6. Consider the two lines whose equations are y = x + a and y = b – x, where a and b are nonzero real numbers. Which of the following statements must be true?
I. If the two lines intersect, then a = b.
II. If a = b, then the two lines intersect on the y-axis.
III. If a = b, then the two lines intersect on the x-axis.
(A) I only
(B) II only
(C) III only
(D) I and II only
7. Which of the following is equivalent to ?
(A)
(B)
(C)
(D)
8. If m ≠ 0, m ≠ 1, and f(x) = mx + b, then which of the following statements concerning the graphs whose equations are y = f(x) + 3 and y = f(x + 3) must be true?
(A) The graphs don’t intersect.
(B) The graphs intersect in one point.
(C) The graphs intersect in two points.
(D) The graphs intersect in more than two points.
9. For how many positive integers, x, does the function have no real values?
(A) 2
(B) 3
(C) 4
(D) Infinitely many
10. If for all real numbers x, h(5 – x)= x2 + x + 1, what is the value of h(9)?
(A) 13
(B) 21
(C) 28
(D) 91
11. A white cube has a volume of 27. If a red circle of radius 1 is painted on each face of the cube, what is the total area of the surface of the cube that is not red?
(A) 27 – 3π
(B) 27 – 6π
(C) 54 – 6π
(D) 54 – 12π
12. Tim’s Tennis Camp is open only to teenagers—all campers must be between 13 and 19 years old, inclusive. Which of the following inequalities can be used to determine if a person who is y years old is eligible to attend the camp?
(A) |y – 13| ≤ 6
(B) |y – 13| ≤ 19
(C) |y – 19| ≤ 13
(D) |y – 16| ≤ 3
13. In the figure above, line segment is perpendicular to side of equilateral triangle ABC. If AB = 12 and BE = 8, what is the area of quadrilateral ABED?
(A)
(B)
(C)
(D)
14. The list price of a certain book is d dollars. Anne bought a copy of the book from an online dealer that offers a discount of 10% off the list price of all books and doesn’t collect sales tax. Beth bought the same book at a bookstore, where the book was on sale for 15% off the list price. However, she had to pay 5% sales tax on his purchase. Which of the following statements is true?
(A) Anne and Beth paid the same price for their books.
(B) Anne paid more than Beth for the book.
(C) Anne paid less than Beth for the book.
(D) Who paid more for her book depends on d, the list price.
15. If the lines whose equations are y = ax + b and x = cy + d are parallel, which statement is true?
(A)
(B)
(C) c = a
(D) c = –a
Grid-in Response Directions
In questions 16–20, first solve the problem, and then enter your answer on the grid provided on the answer sheet. The instructions for entering your answers follow.
First, write your answer in the boxes at the top of the grid.
Second, grid your answer in the columns below the boxes.
Use the fraction bar in the first row or the decimal point in the second row to enter fractions and decimals.
Grid only one space in each column.
Entering the answer in the boxes is recommended as an aid in gridding but is not required.
The machine scoring your exam can read only what you grid, so you must grid-in your answers correctly to get credit.
If a question has more than one correct answer, grid-in only one of them.
The grid does not have a minus sign; so no answer can be negative.
A mixed number must be converted to an improper fraction or a decimal before it is gridded. Enter as 5/4 or 1.25; the machine will interpret 11/4 as and mark it wrong.
All decimals must be entered as accurately as possible. Here are three acceptable ways of gridding
Note that rounding to .273 is acceptable because you are using the full grid, but you would receive no credit for .3 or .27, because they are less accurate.
16. A(1, 1), B(5, 3), and C(5, 9) are three points in the xy-plane. If is a diameter of Circle 1 and is a diameter of Circle 2, what is the slope of the line that goes through the centers of the two circles?
17. If a and b are positive constants and if a(x – y) = b(y – x), what is the value of the ratio ?
18. If c is a real number and if 1 + i is a solution of the equation x2 – 2x + c = 0, what is the value of c?
19. If (a, b) and (c, d) are the two points of intersection of the line whose equation is y = x and the parabola whose equation is y = x2 – 6x + 12, what is the value of a + b + c + d?
20. If for all real numbers x, what is the value of h(3)?
If there is still time remaining, you may review your answers.
MATH TEST (CALCULATOR)
55 MINUTES, 38 QUESTIONS
Turn to Section 4 of your answer sheet to answer the questions in this section.
Directions: For questions 1–30, solve each problem and choose the best answer from the given choices. Fill in the corresponding circle on your answer sheet. For questions 31–38, solve each problem and enter your answer in the grid on your answer sheet.
Notes:
Calculators ARE PERMITTED in this section.
All variables and expressions represent real numbers unless indicated otherwise.
All figures are drawn to scale unless indicated otherwise.
All figures are in a plane unless indicated otherwise.
Unless indicated otherwise, the domain of a given function is the set of all real numbers x for which the function has real values.
REFERENCE INFORMATION
The arc of a circle contains 360°.
The arc of a circle contains 2π radians.
The sum of the measures of the angles in a triangle is 180°.
1. If Wally’s Widget Works is open exactly 20 days each month and produces 80 widgets each day it is open, how many years will it take to produce 96,000 widgets?
(A) fewer than 5
(B) 5
(C) more than 5 but fewer than 10
(D) 10
2. What is the volume, in cubic inches, of a cube whose total surface area is 216 square inches?
(A) 18
(B) 36
(C) 216
(D) 1,296
3. If 2 – 3n ≥ 5, what is the greatest possible value of 2 + 3n?
(A) –1
(B) 1
(C) 4
(D) 5
4. Which of the following statements concerning the equation is true?
(A) The equation has no solutions.
(B) The equation has exactly one solution.
(C) The equation has exactly two solutions.
(D) The equation has infinitely many solutions.
5. If f(x) = x2 – 3x and g(x) = f(3x), what is g(–10)?
(A) 390
(B) 490
(C) 810
(D) 990
Questions 6 and 7 refer to the following table.
Class | Number of Students | Number in Band |
A | 20 | 5 |
B | 30 | 7 |
C | 23 | 5 |
D | 27 | 6 |
E | 25 | 6 |
6. What is the average (arithmetic mean) number of students per class?
(A) 24
(B) 24.5
(C) 25
(D) 25.5
7. Which class has the highest percent of students in the band?
(A) A
(B) B
(C) D
(D) E
8. In a class, 20 children were sharing equally the cost of a present for their teacher. When 4 of the children decided not to contribute, each of the other children had to pay $1.50 more. How much, in dollars, did the present cost?
(A) 50
(B) 80
(C) 100
(D) 120
9. A system of two equations and their graphs are shown above. If (a, b) and (c, d) are the points of intersection of the circle and the parabola, what is the value of a + b + c + d?
(A) –18
(B) –6
(C) 6
(D) 18
10. In the figure above, what is the value of the cosine of angle C?
(A) 0.4
(B) 0.5
(C) 0.6
(D) 0.8
11. For Jen’s birthday, Wes bought her a ring, a bouquet of flowers, and a box of candy, for which he spent a total of $528. If the flowers cost three times as much as the candy, and the ring cost ten times as much as the flowers and candy combined, how much did he pay for the flowers?
(A) $12
(B) $24
(C) $30
(D) $36
Note: Figure not drawn to scale.
12. The diagram above depicts two gears, G1 and G2. Gear G1, which has 48 teeth, turns clockwise at a rate of 60 rotations per second. If gear G2 has 36 teeth, which of the following statements is true?
(A) Gear G2 turns clockwise at a rate of 45 rotations per second.
(B) Gear G2 turns clockwise at a rate of 80 rotations per second.
(C) Gear G2 turns counterclockwise at a rate of 45 rotations per second.
(D) Gear G2 turns counterclockwise at a rate of 80 rotations per second.
Questions 13–15 are based on the information in the following graphs.
All figures are in billions of dollars.
13. The revenue from lottery ticket sales is divided between prize money and the various uses shown in the graph labeled “Proceeds.” In 2009, approximately what percent of the money spent on tickets was returned to the purchasers in the form of prize money?
(A) 23.5%
(B) 50%
(C) 66%
(D) 74%
14. Approximately what percent of the proceeds that went to the states’ General fund would have to be given to the Senior citizen program so that the proceeds for the Senior citizen program and the Cities would be equal?
(A) 0.9%
(B) 9%
(C) 31%
(D) 48%
15. Assume that in 2010 the sales of Lotto were discontinued, and the dollar value of the sales of all other games increased by 10% compared to 2009. If a new circle graph was created to reflect the Lottery Ticket Sales in 2010, which of the following would be closest to the degree measure of the central angle of the sector representing Instant games?
(A) 100°
(B) 150°
(C) 180°
(D) 200°
16. Each week, Alice’s gross salary is $9.00 an hour for the first 40 hours she works and $15.00 an hour for each hour she works in excess of 40 hours. Her net pay is her gross pay less the following deductions: a flat fee of $20 for her contribution to her health insurance; 8% of her gross salary for payroll taxes, and 15% of her gross pay for withholding taxes. Which of the following expressions represents Alice’s net pay in a week that she works x hours where x > 40?
(A) 0.23(15x – 240) – 20
(B) 0.23(15x – 220)
(C) 0.77(15x – 240) – 20
(D) 0.77(15x – 220)
17. If , which of the following is equal to (1 + i)3?
(A) –2 + 2i
(B) 2 – 2i
(C) 4
(D) 4 + 4i
18. If the x-intercepts of the graph of y = 4x2 – 8x + 3 are a and b, what is the value of a + b?
(A) 0.5
(B) 1
(C) 1.5
(D) 2
Questions 19–20 are based on the information in the following graph.
19. If the above circle graph were drawn to scale, then which of the following is closest to the difference in the degree measurements of the central angle of the sector representing Brand C and the central angle of the sector representing Brand D?
(A) 5°
(B) 12°
(C) 18°
(D) 25°
20. The total sales of Coast Corporation in 2005 were 50% higher than in 2000. If the dollar value of the sales of Brand A was 25% higher in 2005 than in 2000, then the sales of Brand A accounted for what percentage of total sales in 2005?
(A) 20%
(B) 25%
(C)
(D) 50%
21. Store 1 is a full-service retail store that charges regular prices. Store 2 is a self-service factory-outlet store that sells all items at a reduced price. In January 2014, each store sold three brands of DVD players. The number of DVD players sold and their prices are shown in the following tables.
Number of DVD Players Sold
Store 1 | Store 2 | |
Brand A | 10 | 30 |
Brand B | 20 | 40 |
Brand C | 20 | 20 |
Prices of DVD Players
Brand A | Brand B | Brand C | |
Store 1 | $80 | $100 | $150 |
Store 2 | $50 | $80 | $120 |
What was the difference between Store 1 and Store 2 in the dollar values of the total sales of the three brands of DVD players?
(A) 80
(B) 140
(C) 330
(D) 1,300
Questions 22 and 23 refer to the figure below, which represents a solid piece of wood being used in the construction of a house. All of the dimensions are in feet.
22. What is the area, in square feet, of the triangular face of the solid?
(A) 24
(B) 48
(C) 50
(D) 80
23. If the density of the wood is 3 pounds per cubic foot and if the weight of the solid is 360 pounds, what is the width, w, in feet, of the solid?
(A) 5.0
(B) 2.5
(C) 2.4
(D) 1.5
Note: Figure not drawn to scale.
24. In the figure above, both triangles are equilateral. If the area of ΔABC is 6 and the area of ΔDEF is 10, to the nearest hundredth what is the ratio of AB to DE?
(A) 0.36
(B) 0.60
(C) 0.75
(D) 0.77
25. If g(x) = (sin x + cos x)2, what is ?
(A) 1
(B) 1.366
(C) 1.866
(D) 2
Questions 26–28 are based on the two graphs below.
26. In which year was the total acreage of farmland in the United States the smallest?
(A) 1940
(B) 1970
(C) 2000
(D) 2010
27. In 2010, the states with the most total acres of farmland were Texas, Montana, Kansas, Nevada, and New Mexico. The acreage in each state (measured in millions of acres) was 130, 61, 46, 46, and 43, respectively. Those five states accounted for approximately what percent of the total farm acreage in the country?
(A) 15%
(B) 25%
(C) 35%
(D) 45%
28. If future projections are that the number of farms in the United States will decrease by 5% from 2010 to 2030 and that the average size of farms will decrease from 2010 to 2030 by the same percent as the decrease from 1990 to 2010, which of the following is closest to the total number of acres of farmland, in millions of acres, in the United States in 2030?
(A) 750
(B) 800
(C) 850
(D) 900
29. Let (h, k) be the center and r the radius of the circle whose equation is x2 + 2x + y2 – 4y + 1 = 0. What is the value of h + k + r?
(A) 1
(B) 2
(C) 3
(D) 4
30. In parallelogram ABCD, each side measures 10. If m∠A = 45°, what is the area of the parallelogram?
(A) 50
(B) 64.6
(C) 70.7
(D) 78.2
Grid-in Response Directions
In questions 31–38, first solve the problem, and then enter your answer on the grid provided on the answer sheet. The instructions for entering your answers follow.
First, write your answer in the boxes at the top of the grid.
Second, grid your answer in the columns below the boxes.
Use the fraction bar in the first row or the decimal point in the second row to enter fractions and decimals.
Grid only one space in each column.
Entering the answer in the boxes is recommended as an aid in gridding but is not required.
The machine scoring your exam can read only what you grid, so you must grid-in your answers correctly to get credit.
If a question has more than one correct answer, grid-in only one of them.
The grid does not have a minus sign; so no answer can be negative.
A mixed number must be converted to an improper fraction or a decimal before it is gridded. Enter as 5/4 or 1.25; the machine will interpret 11/4 as and mark it wrong.
All decimals must be entered as accurately as possible. Here are three acceptable ways of gridding
Note that rounding to .273 is acceptable because you are using the full grid, but you would receive no credit for .3 or .27, because they are less accurate.
31. On a particular map of Long Island, one inch represents a distance of 20 miles. One day, Maurice drove from Hauppauge to Riverhead, which are 1.25 inches apart on that map, at an average speed of 40 miles per hour. How many minutes did his drive take?
32. Michelle participated in a 26-mile marathon that proceeded along a straight road. For the first 20 miles, she ran at a constant pace. At some point she passed a friend who was standing on the side of the road and who was cheering her on. Exactly 54 minutes and 36 seconds later, Michelle passed the 12-mile marker, and 31 minutes and 30 seconds after that, she passed the 15-mile marker. How far, in miles, was it from the starting line to the point where her friend was standing?
Base your answer to Question 33 on the information in the following chart that shows the number of employees at Acme Air-Conditioning in three age groups and the average monthly salary of the workers in each group.
Age Group | Number of Employees | Average Monthly Salary |
Under 35 | 12 | $3,100 |
35–50 | 24 | $3,800 |
Over 50 | 14 | $4,200 |
33. What is the average (arithmetic mean) monthly salary, in dollars, for all the employees?
34. To use a certain cash machine, you need a Personal Identification Code (PIC). If each PIC consists of two letters followed by one of the digits from 1 to 9 (such as AQ7 or BB3) or one letter followed by two digits (such as Q37 or J88), how many different PICSs can be assigned?
35. The base of pyramid 1 is a rectangle whose length is 3 and whose width is 2. The base of pyramid 2 is a square whose sides are 3. If the volumes of the pyramids are equal, what is the ratio of the height of pyramid 1 to the height of pyramid 2?
36. On October 20, 2015, one United States dollar was worth 0.88 euros and one Canadian dollar was worth 0.68 euros. On that date, to the nearest whole number, what is the number of Canadian dollars that could be purchased for 100 United States dollars?
Use the following information in answering Questions 37 and 38.
Every day to go to work, Ed drives the 6.3 miles between Exits 17 and 18 on Route 91, always at a constant rate of 60 miles per hour. At 60 miles per hour, Ed’s car can go 24.3 miles per gallon of gasoline; at 70 miles per hour, Ed’s car can only go 20.8 miles per gallon of gasoline.
37. How much less time, in seconds, would it take Ed to drive those 6.3 miles at 70 miles per hour instead of 60 miles per hour?
38. Last year, Ed drove the 6.3 miles from Exit 17 to Exit 18 a total of 240 times (always at 60 miles per hour). If during the year he paid an average of $3.50 per gallon for gasoline, how much more would it have cost him if he had driven at 70 miles per hour each day? (Express your answer to the nearest dollar, and grid it in without the dollar sign.)
If there is still time remaining, you may review your answers.
ESSAY (OPTIONAL)
Directions: This assignment will allow you to demonstrate your ability to skillfully read and understand a source text and write a response analyzing the source. In your response, you should show that you have understood the source, give proficient analysis, and use the English language effectively. If your essay is off-topic, it will not be scored.
Only what you write on the lined paper in your answer document will be scored–avoid skipping lines, using unreasonably large handwriting, and using wide margins in order to have sufficient space to respond. You can also write on the planning sheet in the answer document, but this will not be evaluated–no other scrap paper will be given. Be sure to write clearly and legibly so your response can be scored.
You will be given 50 minutes to complete the assignment, including reading the source text and writing your response.
Read the following passage, and think about how the author uses:
Evidence, such as applicable examples, to justify the argument
Reasoning to show logical connections among thoughts and facts
Rhetoric, like sensory language and emotional appeals, to give weight to the argument
“Think left and think right and think low and think high.
Oh, the thinks you can think up if only you try.”
—Dr. Seuss
1 It was my third year teaching English at Rosinburgh High when the freckled, garrulous Emily of the second row interrupted my comma lesson with an abrupt question, “Mrs. Jensen, why do you write?” I considered telling Emily to stay after class if she wished to converse off topic. Yet, the abridged somnolence in the farthest row and my romantic inclinations to a teaching career somewhat like that of Julia Roberts in Mona Lisa Smile—both sparked by the most recent inquisition—conspired my actual response: “Why does one listen to music? Or dance? Or look to the stars?” Amusement, surely. Communication and complex stimulation, absolutely. But mostly, I write in the name of indomitable creativity. I am of the distinct opinion that creativity is the most essential ingredient of erudition, expression, and future success, and being so, must be encouraged inside the classroom.
2 Creativity, contrary to popular belief, is not limited to the milieu of the elite, pedantic, or particularly adept; it is not reserved for the Picassos, the Bachs, or the Austens. Rather it is a gift bestowed to each and every one of us that should be maintained and even bolstered. The failure to include it in the public curriculum is a failure to society it in its entirety, as it indisputably dissuades curiosity, exploration, and activism. Emily, what if I told you that creativity is behind every piece of art, every new electronic, and every scientific discovery? What if I told you progress would be simply unfeasible without it? If you question its innateness, a moment’s reflection on the sublime imagination of any child you have ever come into the briefest contact with will assure you otherwise.
3 As for its consequence, let us first consider personal fulfillment. Where would you suppose yourself most happy: in a desultory, routine job or an engaging, challenging career? While the question may seem puerile, its implications are far from trivial. The number one reason for unhappiness in adults is hating their jobs, and LinkedIn places “not being challenged” and “not feeling valued” at the top of the list for workplace dissatisfaction. If we can generalize creativity to the extent Google does, i.e., to the generation and contemplation of new ideas to create something perceived as valuable, then we can arrive at the need for it in our workplaces and domestic spheres. Simply put, humans need creativity to feel gratification. You may venture that the heretic without imagination is yet to be born, but I would add that the contented without imagination is just as uncommon.
4 Shall we move on to its effluence in a broader sense? Higher creativity preludes greater innovation and, thus, success and progress. In fact, a 2010 IBM survey ranked creativity as the single most important factor in corporate success. Let us ruminate for a moment on medical cures, energy alternatives, and space exploration—the need for creativity extends far beyond the reclusive artist indeed. Perhaps your aspirations fall short of those of Steve Jobs or Shinya Yamanaka; still, you need to bring problem-solving and critical thinking into your career for the sake of efficiency, diversification, and job security. It is what allows you to see the world as it is, exercise your interest in all that surrounds you, and bring fresh thinking to your circumstance. After all, it was Albert Einstein who said, “Imagination is more important than knowledge.”
5 I have convinced you of creativity’s import, but my rambling will seem only didactic if I neglect to address the ways that creativity can be nourished within the education system. You’ll be surprised to learn how easily it can be incorporated into everyday scholarship. Schools, and teachers more specifically, can cultivate creativity by encouraging questioning and debate. The why’s and the how’s are a good place to start. Similarly, classrooms can make a habit of identifying problems and brainstorming solutions: the what’s good, what’s bad, and how could it be improved. A next step is in inspiring alternative problem solving—these are the beloved what if’s. Promoting creativity begins with the generating of new knowledge, in lieu of the passing on of existing knowledge solely. It may be unclear to Emily why I write, particularly when I have so little success at it, but it is apparent to me how valuable her curiosity is, and how paramount that it be nurtured rather than dulled.
Write a response that demonstrates how the author makes an argument to persuade an audience that creativity should be taught in schools. In your response, analyze how the author uses at least one of the features from the essay directions (or features of your own choosing) to develop a logical and persuasive argument. Be certain that your response cites relevant aspects of the source text.
Your response should not give your personal opinion on the merit of the source text, but instead show how the author crafts an argument to persuade readers.
ANSWER KEY
Practice Test 1
Section 1: Reading
1. C
2. A
3. B
4. C
5. B
6. A
7. B
8. A
9. B
10. B
11. C
12. B
13. D
14. B
15. B
16. A
17. A
18. C
19. C
20. D
21. D
22. D
23. C
24. C
25. B
26. B
27. D
28. B
29. C
30. D
31. C
32. C
33. D
34. A
35. B
36. C
37. D
38. C
39. B
40. B
41. A
42. C
43. B
44. B
45. A
46. A
47. A
48. D
49. C
50. C
51. A
52. C
Number Correct_____
Number Incorrect_____
Section 2: Writing and Language
1. C
2. A
3. D
4. C
5. B
6. D
7. A
8. B
9. A
10. C
11. A
12. C
13. C
14. A
15. B
16. A
17. D
18. A
19. D
20. B
21. D
22. A
23. B
24. A
25. B
26. A
27. C
28. D
29. D
30. D
31. B
32. C
33. A
34. C
35. B
36. C
37. A
38. D
39. B
40. A
41. D
42. D
43. B
44. C
Number Correct_____
Number Incorrect_____
Section 3: Math (No Calculator)
1. A
2. D
3. C
4. D
5. B
6. B
7. A
8. A
9. B
10. A
11. C
12. D
13. B
14. B
15. B
16. 2
17. 1
18. 2
19. 14
20. 3
Number Correct_____
Number Incorrect_____
Section 4: Math (No Calculator)
1. B
2. C
3. A
4. A
5. D
6. C
7. A
8. D
9. A
10. D
11. D
12. D
13. C
14. C
15. D
16. C
17. A
18. D
19. C
20. C
21. D
22. B
23. B
24. D
25. C
26. D
27. C
28. B
29. C
30. C
31. 37.5
32. 6.8
33. 3744
34. 8190
35. 3/2 or 1.5
36. 129
37. 54
38. 37
Number Correct_____
Number Incorrect_____
SCORE ANALYSIS
Reading and Writing Test
Section 1: Reading
Section 2: Writing
To find your Reading and Writing test scores, consult the chart below: find the ranges in which your raw scores lie and read across to find the ranges of your test scores.
To find the range of your Reading and Writing Scaled Score, multiply (C) by 10.
Test Scores for the Reading and Writing Sections
Reading Raw Score | Writing Raw Score | Test Score |
44–52 | 39–44 | 35–40 |
36–43 | 33–38 | 31–34 |
30–35 | 28–32 | 28–30 |
24–29 | 22–27 | 24–27 |
19–23 | 17–21 | 21–23 |
14–18 | 13–16 | 19–20 |
9–13 | 9–12 | 16–18 |
5–8 | 5–8 | 13–15 |
less than 5 | less than 5 | 10–12 |
Math Test
Section 3:
Section 4:
Total Math raw score: (D) + (E) = ___________
To find your Math Scaled Score, consult the chart below: find the range in which your raw score lies and read across to find the range for your scaled score.
Scaled Scores for the Math Test
Raw Score | Scaled Score |
50–58 | 700–800 |
44–49 | 650–690 |
38–43 | 600–640 |
32–37 | 550–590 |
26–31 | 500–540 |
20–25 | 450–490 |
15–19 | 400–440 |
11–14 | 350–390 |
7–10 | 300–340 |
less than 7 | 200–290 |
ANSWERS EXPLAINED
Section 1: Reading Test
1. (C) Substitute the answer choices in the original sentence. The sergeant is a person who might have been a deputy sheriff before he joined the army—that is, in his civil or nonmilitary life.
2. (A) Paragraph 1 presents a general picture of the man on the bridge, the executioners and the offi-cer standing nearby, the sentinels at the far ends of the bridge. Cinematically, it is like a wide-angle shot of the whole panorama. Paragraph 2 takes a closer look at the man, examining his clothes, his face, his expression. It is as if the camera has moved in for a close-up shot.
3. (B) You can use the process of elimination to answer this question. Was the man awaiting hanging innocent of any criminal intent? No. He was willing to go along with the idea that all was fair in love and war, and would willingly perform a criminal act (assaulting a sentinel and burning down a bridge). Choice (A) is incorrect. Was the man awaiting hanging an unlikely candidate for execu-tion? Possibly. Keep choice (B) in mind as you consider the other choices. Was the man await-ing hanging a victim of mistaken identity? No. He was caught in the act of attempting to burn down the bridge. Choice (C) is incorrect. Was the man awaiting hanging purposely assuming a harmless demeanor? Nothing in the passage suggests that he was putting on the appearance of being harmless. Choice (D) is incorrect. Only choice (B) is left. It is the correct answer.
4. (C) To have one’s neck “in the hemp” is to have one’s neck in a noose, a rope made out of hemp. The author’s comment that the man “had a kindly expression that one would hardly have expected in one whose neck was in the hemp” sug-gests that he is an unlikely candidate for execution and that some unusual circumstances must have brought him to this fate.
5. (B) In calling the military code “liberal” because it doesn’t exclude members of the upper classes from being executed, the author is being highly ironic. Generally, people would like regulations to be interpreted liberally to permit them to do the things they want. Here, the liberal military code is permitting the man to be hanged. Clearly, the gentleman facing execution would have preferred the code to be less liberal in this case.
6. (A) Farquhar agrees readily with the saying that all is fair in love and war. This implies he is willing to use underhanded or unfair methods to support his [the Southern] cause.
7. (B) Look at the context in which the word consistent occurs. “(N)o adventure [was] too perilous for him to undertake if [it was] consistent with the character of a civilian who was at heart a soldier.” Farquhar has no objection to performing humble errands or undertaking dangerous tasks as long as these tasks are appropriate to someone who sees him-self as a sort of “undercover soldier,” a secret agent of the Confederacy. Anything he does must be consistent or compatible with his image of himself in this role.
8. (A) The fact that Mrs. Farquhar is married to a man “ardently devoted to the Southern cause,” together with her readiness to fetch water for a Confederate soldier, suggests some degree of sympathy on her part for the Confederate cause. Choice (B) is incorrect. Mrs. Farqu-har’s action, in hospitably fetching water “with her own white hands,” contra-dicts the idea that she is too proud to perform menial tasks. Choices (C) and (D) are also incorrect. There is nothing in the passage to suggest either of them.
9. (B) The assertion that Mrs. Farquhar “was only too happy” to fetch water for a soldier wearing the grey uniform of the Confederate army provides strong evidence that she is sympathetic to the Confederate cause.
10. (B) Farquhar wishes to prevent the Yankee advance. To do so, he must somehow damage the rail-road, its bridges, its tunnels, or its trains. The soldier tells him that some highly flammable driftwood is piled up at the base of the wooden railroad bridge. Clearly, it would make sense for Farquhar to try to set fire to the driftwood in an attempt to burn down the bridge.
11. (C) The scout is a Yankee soldier disguised as a member of the enemy. By coming to the Farqu-hars’ plantation in Confederate disguise, he is able to learn they are sympathetic to the enemy. By telling Farquhar of the work on the bridge, stressing both the lack of guards and the abundance of fuel, he is tempting Farquhar into an attack on the bridge (and into an ambush). The scout’s job is to locate potential enemies and draw them out from cover. The concluding sen-tence thus establishes that Farquhar has been entrapped into taking an unwise action, an action that will lead to his execution at Owl Creek bridge.
12. (B) President Roosevelt mentions two types of people who came to America seeking freedom: “some of high degree, but mostly plain people.” The people of high degree were members of the upper classes, those set apart by aristocratic birth or social position; the others were plain, ordinary people.
13. (D) Roosevelt describes the American spirit as “the product of centuries.” In other words, the hope of freedom and the love of liberty go back for centuries; America’s democ-ratic ideal has deep-seated (firmly established) historical roots.
14. (B) Choice (B) clearly supports the contention that the American belief in freedom “has deep-seated historical roots.” It pointedly asserts that the “democratic aspiration is no mere recent phase in human history” and goes on to mention the Magna Carta or Great Charter of 1215 as a specific example of a historic document that embodies the spirit of democracy.
15. (B) Look at the context in which these documents are mentioned. “Its vitality was written into our own Mayflower Compact, into the Declaration of Independence, into the Constitution of the United States, into the Gettysburg Address.” To what does the phrase “Its vital-ity” refer? The answer appears two paragraphs earlier, in the opening sentence “The democratic aspiration is no mere recent phase in human history.” The democratic aspiration is Roosevelt’s theme, the subject he is discussing. According to Roosevelt, the vitality and strength of democratic aspiration were written into America’s founding documents. Thus, he clearly is citing these documents as examples of expressions of the democratic aspiration.
16. (A) In lines 36–39, Roosevelt asserts that Americans “have moved forward con-stantly and consistently toward an ideal which in itself has gained stature and clarity with each gen-eration.” It is his optimistic contention that we have been coming closer and closer to reach-ing that ideal and that the democratic spirit has grown stronger. However, he acknowledges that “we still have far to go” before we reach that ideal: the existence of both “undeserved poverty” and “self-serving wealth” stands in the way of democracy. In other words, economic injustices must be addressed before democracy can pre-vail.
17. (A) In his 1789 First Inaugural Address, Washington was directing his words to his contemporaries. In other words, he was addressing his words to them, speaking to them in a formal way. To Roosevelt, Washington’s words sounded prophetic, as if he addressed his words to the people of 1941.
18. (C) Repetition is a common, yet effective literary device that strengthens the power of the point be-ing made. One website of popular literary terms describes the effect of repetition as follows: “The aura that is created by the usage of repetition cannot be achieved through any other device. It has the ability of making a simple sentence sound like a dramatic one. It enhances the beauty of a sentence and stresses on the point of main significance.” Thus, the main effect of the repetition of the phrase “It speaks to us” is to create a dramatic tone, em-phasizing the point being made and adding to its emotional impact.
19. (C) What experiment was “intrusted to the hands of the American people”? Consider the context. Washington was writing an address that he was to deliver on his inauguration as presi-dent of the United States, a new nation founded upon democratic principles. The experiment en-trusted to the American people was the radical experiment of democratic government.
20. (D) Washington’s “experiment intrusted to the hands of the American peo-ple” is the republican model of government, whose preservation he urges. It is this same model of government that Roosevelt urges Americans to defend when he asserts that “our strong purpose is to protect and to perpetuate the integrity of democracy.”
21. (D) As we know from the introduction to the passage, Roosevelt delivered his Third Inaugural Ad-dress in January of 1941, at a time when Britain and its allies were hard pressed by Hitler’s forces but the United States had not yet gone to war. Nonetheless, it was clear that democracy was under attack and that America was facing “great perils never before encountered.” The passage’s final paragraphs emphasize the need to preserve the sacred fire of liberty. The president exhorts his listeners “to protect and to perpetuate the integrity of democ-racy.” Thus, it is reasonable to conclude that a major goal of Roosevelt in making this speech was to inspire the American people to defend the cause of freedom in dangerous times.
22. (D) Step by step, the author traces the course of a surgical procedure, from the initial grasping of the scalpel through the opening incision to the eventual sensory exploration of the internal organs. In doing so, he is describing a process. Choice (A) is incorrect. Although in the course of the passage the author occasionally defines a term (for example, the term fascia), the pas-sage, taken as a whole, describes the process of surgery; it does not define a term. Choice (B) is incorrect. The passage does not provide an example of a particular method of surgery; instead, it describes the process of surgery. Choice (C) is incorrect. This is not a lesson, instructing novice sur-geons in the steps they should take to perform a successful abdominal surgery; it is a vivid descrip-tion of the process of surgery.
23. (C) As the surgeon draws the knife across the skin, it leaves a thin Line
of blood in its wake (path or track passed over by a moving object). Choices (A) and (B) are incorrect; nothing in the opening paragraph supports either choice. Choice (D) is incorrect. The darting knife is followed by the fine wake of red; the knife blade is not the wake of red.
24. (C) The darting knife is followed by a wake of red. If the meaning of the simile is still unclear at this point, the subsequent sentence, in which the parted flesh reveals yellow blobs of fat below, is a clue that the “fine wake of red” is a bloody incision made by the knife. Choices (A) and (B) are incorrect. They describe the way to hold the knife, not the effect of the knife as it cuts its way through the patient’s skin. Choice (D) is incorrect. It has nothing to do with the knife’s wake or trail of red.
25. (B) To part the flesh is to split apart or separate the skin, cutting it apart with the knife. Choice (A) is incorrect. Although in some contexts “parts” means leaves, as in “parting from someone at the train station,” that is not how it’s used here. Choice (C) is incorrect. Although in some contexts “parts” means surrenders (“parting with hard-earned cash”), that is not how it is used here. Choice (D) is incor-rect. Although in some contexts “parts” means distributes (“parting an estate into shares”), that is not how it’s used here.
26. (B) The simile “like children absorbed in a game” indicates that, in this context, “engaged” means engrossed or deeply involved. Choice (A) is incorrect. Al-though in some contexts “engaged” means betrothed or pledged to marry, that is not how it is used here. Choice (C) is incorrect. Although in some contexts “engaged” means hired (“engaged as a contractor”), that is not how it is used here. Choice (D) is incorrect. Although in some contexts “engaged” means embattled (“engaged forces”), that is not how it is used here.
27. (D) Primitive drawings of buffalo and other wild beasts still exist in caves in which prehistoric humans dwelled. Thus, one might expect to find them in a cavity described as “a primitive place.” Choice (A) is incorrect. In “a primitive place,” one might expect to find primitive drawings; one wouldn’t necessarily expect to find them in an art gallery. Choices (B) and (C) are incorrect. Nothing in the passage suggests that one might expect to find drawings of buffalo in either a zoological display or a Western film.
28. (B) The colors of the internal organs are secret because, until the peritoneum is opened and the world’s light illuminates the abdominal cavity, the internal organs cannot be seen. In other words, the colors of the internal organs normally are hidden from sight.
29. (C) The author looks on his work as a surgeon as if it were a priestly vocation. The first hint of this comes in lines 32–34: “the litany of monosyllables with which one prays his way down and in: clamp, sponge, suture, tie, cut.” The one-word requests that the surgeon makes for a clamp or a sponge are like a litany, a form of prayer made up of a series of invocations or peti-tions that are usually led by the clergy. The surgeon “prays his way down and in.” Clearly, the author is making use of priestly or religious imagery. Choice (A) is incorrect. Al-though the author likens the abdominal cavity to a cavern, he primarily describes the surgery in reli-gious terms. Choice (B) is incorrect. The author never contrasts abdominal surgery with other types of surgery. Choice (D) is incorrect. The author never evokes or suggests the patient’s emotions; if anything, he evokes the surgeon’s emotions.
30. (D) In these two sentences, the author makes explicit his sense of surgery as a religious rite, an impression he continues to develop in the subsequent paragraph (“the priestliness of my profession has ever been impressed on me”).
31. (C) Consider the various descriptive passages in which the author explores the formerly hidden or-gans and tissues now exposed to view through surgical intervention. “The peritoneum, pink and gleaming and membranous, bulges into the wound.” “An arc of the liver shines high and on the right, like a dark sun. It laps over the pink sweep of the stomach, from whose lower border the gauzy omentum is draped, and through which veil one sees, sinuous, slow as just-fed snakes, the indolent coils of the intestine.” Peritoneum, liver, stomach, omentum, intes-tine: the author names them, evokes their appearance in a brief descriptive phrase (“peritoneum, pink and gleaming and membranous,” “gauzy omen-tum,” “indolent coils of the intestine”). However, he does not bother to define these anatomical terms. Instead, he apparently assumes that his readers will be familiar with the organs and tissues to which he refers. Choice (A) is incorrect. The author freely goes into vivid detail about surgical procedures. He would be unlikely to be as free in his imagery if he assumed that his readers had qualms about reading descriptions of surgery. Choice (B) is incorrect. The au-thor carefully describes how to hold a scalpel; he would not do so if he assumed his readers were already adept at handling surgical tools. Choice (D) is incorrect. Nothing in the passage suggests that the author assumes his readers have undergone surgery.
32. (C) Throughout Passage 1, the author constantly emphasizes the lengthy time period his book will cover. In the passage’s opening sentence he refers to “the shifting relationship be-tween the press and the presidency over nearly two centuries”; later he refers to “the relationship of the presidents to the press since George Washington’s first term.” The author is “surveying nearly two centuries of this relationship.” In other words, he is presenting an overview of a relationship. What sort of relationship is it? It is one of “mutual . . . recriminations and mistrust.” There has been one main constant in it, “the dissatisfaction of one with the other.” In other words, it is an inherently conflicted relationship. Not only that, but this relationship “has been increasingly compli-cated by the changing nature of the presidency, by the individual nature of presidents, by the rise of other media, especially television, and by the growing complexity of beliefs about the function of both press and government.” The inherently conflicted relationship clearly faces new challenges and will continue to do so as our institutions and technology continue to change. The correct answer is choice (C).
33. (D) The first paragraph of the passage says that the administration of every president has ended with “recriminations and mistrust.” Presidents, like everyone else, hate to be criticized in public. Therefore, they all have experienced animosity involving themselves and the press. Choice (C) is incorrect. The first paragraph states that, while the initial stage of the relationship be-tween the press and the president may seem harmonious (“beginning with mutual protesta-tions of good will”), the relationship ends in antagonism. The paragraph never suggests that periods of antagonism and harmony alternate.
34. (A) If the record of every administration has begun with the press and the presidency claim-ing to feel goodwill toward one another and has ended with them blaming and mistrusting one an-other, then clearly all American presidents have experienced mutual animosity (ill will) involving themselves and the press.
35. (B) To be reduced to simple terms is to be simplified or boiled down to its essential, basic nature. Choices (A), (C), and (D) are incorrect. Although reduced can mean decreased (“reduced speed”), marked down (“reduced prices”), or demoted (“reduced in rank”), it is not how the word is used here.
36. (C) The author advises the reader to (lines 41–42) “be careful not to view the past in terms of our own times.” This advice is an axiom (a statement or proposition regarded as being established, accepted, or self-evidently true) of professional historians. As a professional his-torian, he gives the reader this advice because readers today cannot fully grasp the significance these actions had in their own time.
37. (D) To use present standards to judge the actions of past presidents “is to violate historical context.” Because the words “press,” “government,” and “presidency” have changed in meaning, because our lives today differ so greatly from the lives of people in earlier times, we cannot fully understand the significance of presidential actions two centuries ago.
38. (C) The opening sentence of the final paragraph concludes with the clause “but the facts are quite different.” Many Americans believe that the colonists immediately established a free society. The author says that this belief is incorrect. Thus, he is trying to correct a misconcep-tion or mistaken idea.
39. (B) The open questioning to which the author refers (line 107) is the unrestricted, no holds barred questioning that the president faces in modern solo press conferences. Choices (A), (C), and (D) are incorrect. Although “open” can mean receptive (“open to sug-gestions”), unconcealed (“open carry”), or vulnerable (“open to abuse”), that is not how it is used here.
40. (B) Use the process of elimination to answer this question. Did President Clinton hold more solo press conferences than President George H. W. Bush did? No. President Clinton held 62 solo press conferences; President George H. W. Bush held 83 solo press conferences. Choice (A) is incorrect. Did Presidents Clinton and George W. Bush hold a far higher percentage of joint press conferences than either of their predecessors did? This seems to be correct. President Clinton’s percent-age of joint press conferences was 67.9%; President George W. Bush’s percentage of joint press conferences was 77.3%. Neither of their predecessors came even close. Choice (B) is most likely the correct answer. Quickly scan the remaining choices to see whether you can find a better answer than choice (B). Choice (C) asks you to draw a conclusion that is unsupported by the data in the graph. True, the graph provides data showing President Reagan held no joint press confer-ences; however, it provides no information to indicate why he did this. Choice (C) is incorrect. Choice (D) is also incorrect. Unlike his son, President George H. W. Bush held fewer joint press conferences than solo sessions.
41. (A) Passage 1 describes a phenomenon. This phenomenon is the dissatisfaction of the press and the president with each other. Passage 1 describes it as a constant, that is, an un-changing factor. Passage 2, however, focuses on changes in this relationship. It does not describe the situation as static. Instead, it emphasizes the “increasingly contentious, adversar-ial” nature of the relationship and suggests that this increased aggressiveness from the press may have brought about the shift from solo press conferences, during which the president is more open to direct hostility, to joint press conferences, in which he has more control of the situation. Thus, in focusing on the increases in press aggressiveness and on the changes in the structure of presidential press conferences, Passage 2 denies the static nature of the phenomenon described in Passage 1.
42. (C) Both authors make a point of the adversarial nature of the relationship between the press and the president. This inherent antagonism is at the heart of the relationship and is likely to influence the actions of both the press and the president for years to come.
43. (B) Choice (B) is correct. To describe the debate over fracking or hydraulic fracturing as “fractious” is to make a play on the words “fractious” and “fracturing.” The author chooses the adjective “fractious” because it begins like “fracturing.” The similarity in sound between the two words strengthens the sentence’s effect. In contrast, consider the effect of this slight change on the opening sentence: “To call it a heated debate is an understatement.” Lacking the word play, the revised sentence feels a bit flat. Choice (A) is incorrect. A metaphor is a figure of speech in which a term or phrase is applied to something to which it is not literally applicable in order to sug-gest a resemblance, as in “My boss is such a bear today.” Choice (C) is incorrect. It is no exaggeration to say that the debate over fracking has become heated or fractious. Hydraulic fracturing is a controversial subject, and the discussion about it is contentious. Choice (D) is incorrect. A counterargument is an argument put forward to oppose an idea or theory developed in another argument. It has nothing to do with the author’s word choice here.
44. (B) Choice (B) is correct. A game changer is an event, idea, or procedure that brings about a signifi-cant shift in the current way of doing something. By calling fracking-related gas recovery a game changer, fracking’s supporters are asserting that fracking is going to radically alter natural gas production. Choice (A) is incorrect. In calling fracking a game changer, its proponents do not assert that fracking has no foreseeable negative consequences. Instead, they assert that its positive benefits (reduced dependence on foreign sources of energy, new jobs, relatively clean energy, etc.) strongly outweigh its possible drawbacks. Choice (C) is incorrect. In calling fracking a game changer, its proponents are not asserting that they fail to take it seriously. Choice (D) is incorrect. Although later portions of the passage mention a need for regulation, fracking’s proponents say nothing about any need for active federal regulation.
45. (A) Choice (A) is correct. In lines 6–14, the author lists the following points made by frack-ing’s supporters:
1. Compared to natural gas recovered by drilling oil wells, natural gas recovered through fracking is inexpensive (it “is cheap”).
2. Compared to natural gas recovered by drilling oil wells, natural gas recovered through fracking is relatively free from pollutants or unpleasant substances (it is “relatively clean”).
3. The United States contains an abundance of natural gas that can be recovered through fracking. (“America is brimful of the stuff.”)
To have access through fracking to an abundant supply of inexpensive, relatively clean natural gas would change our methods of natural gas production radically. It would change the entire oil industry. Indeed, it is doing so. None of the remaining choices provide evidence in support of the assertion that fracking is a game changer.
46. (A) In lines 15–24, the author relates the public’s fears about the dangers of fracking. Media reports of fracking-triggered earthquakes and fracking-caused environmental contamination fuel these fears. In the paragraph immediately following, the author depicts the reaction these fears have produced in the political arena. She mentions protests and legislative attempts to halt or ban fracking in several states. Her discussion of fracking legislation thus describes specific re-sponses (marches, moratoriums, policy changes) to concerns raised in the previous para-graph.
47. (A) Choice (A) is correct. An open question is a matter that has not yet been decided, an issue that remains unresolved. Choice (B) is incorrect. Although “open” can mean vulnerable, as in the welfare system’s being “open to abuse,” that is not the sense in which it is used here. Choice (C) is incorrect. Although “open” can mean accessible, as in a school program’s being “open to all stu-dents,” that is not the sense in which it is used here. Choice (D) is incorrect. Although “open” can mean ajar, as in a door’s being “left open,” that is not the sense in which it is used here.
48. (D) Use the process of elimination to answer this question. Choice (A) is incorrect. Nothing in the passage suggests that its author advocates or supports technological innovations such as fracking; she merely reports the opinions of fracking’s advocates. Choice (B) is incorrect. Nothing in the passage suggests that its author either opposes or supports regulatory oversight, whether point-less or not; she merely describes legislative attempts to regulate fracking. Choice (C) is incorrect. Nothing in the passage suggests that its author is a concerned legislator; she merely recounts the actions taken by legislators regarding fracking. Only choice (D) is left. It is the correct answer. The author is an observer striving to be objective and to present a balanced account.
Remember to read the italicized introduction. Often it contains useful information. Here, the itali-cized introduction indicates that the passage comes from a popular science magazine, Science News. Such magazines have the task of presenting current research findings in an objective, unbiased manner, weighing both sides of an argument rather than arguing the merits of a particular claim.
49. (C) The author acknowledges that the dangers of fracking are real. She also acknowledges that the potential energy payoff is real as well. She is striving to present both sides of the argument objec-tively.
50. (C) The solid line on the graph vividly depicts a sudden, marked jump in the number and frequency of earthquakes from about 2009. This accentuates or emphasizes the increasing frequency of earthquakes in the region. Choice (B) is incorrect. The graph plots the increased frequency of earthquakes, not their increased magnitude. Choices (A) and (D) are incorrect. Nothing in the pas-sage supports either answer.
51. (A) In stating that the change in the frequency and number of earthquakes was pronounced, Ells-worth is asserting that it was marked or particularly noticeable. Choices (B), (C), and (D) are incorrect. Although “pronounced” can mean declared (“pronounced dead”), decided on (“pronounced on innocence or guilt”), or articulated (“correctly pronounced words”), that is not how it is used here.
52. (C) Ellsworth indicates that researchers lack pre-quake data (lines 93–95). He advises re-searchers to collect seismic information about current and potential drilling sites. He concludes the paragraph by stating explicitly that “(t)here are many things we don’t under-stand.” All these comments in the passage’s concluding paragraph suggest that Ellsworth looks on current hypotheses about connections between the recent increases in earth-quakes and the start of the fracking boom as based on insufficient knowledge. Geologists simply don’t know enough about what actually occurs during fracking to be able to properly test their hypotheses about fracking’s possible effects on the increasing frequency of earth-quakes.
Section 2: Writing and Language
1. (C) The first sentence of the paragraph states that Modernism is characterized by the “complete rejection” of traditions and values, which is consistent with a “rebuff” of conventional morality. Moreover, the remainder of the paragraph mentions moral relativism, which further solidifies the notion of a “rebuff” of mainstream values. Choice (A) is not correct because it is the opposite of an embrace. Choice (B) is not right because it can be reasonably inferred that Fitzgerald must have understood conventional morality since he skillfully wrote about matters concerning it. Choice (D) is incorrect because its connotation is too positive.
2. (A) The writer uses an interesting turn of phrase to state that Bernice does not listen to (entertain) or find amusing (be entertained by) Marjorie’s social activities. Choice (B) does not work be-cause a transitional word would be needed after “entertainment.” Choices (C) and (D) result in nonsensical meanings.
3. (D) The underlined portion refers to the female Bernice, so “herself” is appropriate. The other options are not consistent with a third-person singular female.
4. (C) ”To mock” is parallel with the earlier “to embody” in the sentence and concisely expresses the intended idea. Choices (A) and (B) are too wordy. Choice (D) is not parallel to the earlier phrasing.
5. (B) ”Who” is correct since it stands for a subject that is human. Choice (A) is used in reference to objects. Choice (C) shows possession. Choice (D) means “who is.”
6. (D) This sentence needs to come before the third sentence, which starts with “Despite her good looks. . . .” This sentence provides a transition between a description of Marjorie and a contrasting description of Bernice. The other placements are illogical as they would not allow for a clear transition between the descriptions of the two characters.
7. (A) Two eras are being compared, so “between” is the best choice. Choice (B) is wrong because “among” is used for a comparison of three or more things. “Contrast for,” which is choice (C), and “Contrast on,” which is choice (D), are not idiomatically correct.
8. (B) ”While” is the only option that provides a contrast within the sentence between how Marjorie is preferred socially and her rudeness.
9. (A) When adjectives have to be ordered a certain way to provide a logical meaning, there should be no commas separating them. In this case, it only makes sense to say “few short sen-tences,” not “short few sentences,” making choice (A) the only viable option. Choices (B) and (D) change the meaning, and choice (C) has an unnecessary comma.
10. (C) The last sentence of the paragraph makes this choice the most clear, stating that both Bernice and Marjorie are quite superficial. So characterizing them as having “false facades” is most logical. With their vengeful dramatics, they are far from being “kind-hearted” as in choice (A). Choice (B) is incorrect since they are not revolutionary ideologues but, instead, more decadent. Choice (D) is not the right answer because being frivolous and pious is contradictory.
11. (A) In choice (A), the colon comes after a complete sentence right before the flaws are clarified and the comma comes before the transitional “while.” Choice (B) results in a run-on sen-tence. Choice (C) has an unnecessary comma after “lifeless.” Choice (D) has an un-necessary comma after “manner.”
12. (C) The first part of the sentence states that journalism gathers and distributes news in a wide vari-ety of ways, and choice (C) gives specific examples of the technology that does this. Choices (A), (B), and (D) are irrelevant to the first part of the sentence.
13. (C) This choice places a comma after the introductory dependent clause ending in “media” and puts a colon before a clarification of how the field is optimistic. Choice (A) lacks a necessary comma after “media” and leads to a run-on sentence. Choice (B) leads to a run-on. Choice (D) puts a semicolon between a subject and a verb, which should not be separated.
14. (A) ”But also” follows “not only” when making a statement like “not only this but also that.” None of the other options works with this idiomatic phras-ing.
15. (B) The current paragraph emphasizes that journalism has undergone major changes, while the following paragraph delves more deeply into concrete explanations of these changes. So choice (B) makes the most sense because it provides both a conclusion to the current paragraph and a transi-tion into the topic of the next. Choices (A) and (C) contradict the information presented in the next paragraph. Choice (D) speaks more to the quality of journalism than to its overall popularity.
16. (A) The sentence is stating two things that do not happen, so saying “doesn’t” in conjunction with “nor” makes sense. Choice (B) shows cause and effect. Choice (C) shows a direct connection between two ideas. Choice (D) shows contrast.
17. (D) This choice correctly does not have an apostrophe after “departments” because this word is functioning as the subject, not as a possessive adjective. Choices (A) and (B) incorrectly have apostrophes after “departments.” Choice (D )is also correct because “ensuring” means to “make sure,” which fits the context. “Assure” means to “reassure,” and “insure” has to do with financial transactions.
18. (A) The previous sentence refers to “leading journalism departments,” so a sentence about the number one journalism department in the country is a logical connection. The other choices may very well give interesting and factual information about this school, but they are not directly connected to the previous sentence.
19. (D) This choice correctly places commas around the parenthetical phrase. Choice (A) has a comma at an awkward point, choice (B) lacks the necessary pauses, and choice (C) is too choppy.
20. (B) The adjective “endless” is needed to modify the noun “effects.” Also, “affect” is generally a verb, and “effect” is generally a noun. The incorrect options either use the adverb “endlessly” and/or use the verb “affect.”
21. (D) According to the graph, the number of journalism-related job openings has gone up, then down, and then up again in recent years. This variation is best described as a “fluctuation.” Choice (A) is incorrect because since the passage was written in 2015, there has not been a steady increase in demand for journalists in recent years, given the big drop from 2006 to 2009. Choice (B) is not right because in recent years, the number of jobs available increased. Choice (C) is not correct because the level of employment has gone up and down, not remained steady.
22. (A) To “adapt” is to make something suitable, and to “adopt” is to make something one’s own. In this case, the field of journalism is making gradual changes in order to adjust to technological advances, so “adapt” makes sense. Choices (C) and (D) are therefore incorrect. The paragraph is in the present tense, so “is adapting” works (choice (A)) and “was adapting” (choice (B)) does not.
23. (B) Giving an approximation of the years is the most precise option. Choices (A), (C), and (D) are too vague.
24. (A) This choice gives necessary breaks between all of the listed items and also has a break after the clarifying phrase “most recently.” Choices (B), (C), and (D) all change the original meaning because of their comma placements or lack thereof.
25. (B) To convey giving a high priority to making the Parthenon extravagant, the phrase “sparing no expense” is the best choice. It indicates that the Greeks were willing to put as many economic resources as needed into finishing the Parthenon in an extravagant manner. Honesty and judiciousness, choice (A), do not necessarily relate to making the Parthenon extrava-gant. Although “artistic patience” and the use of “architectural tech-niques” (choices (C) and (D)) could be loosely related to completing the Parthenon in an ex-travagant fashion, these do not convey that spending money was a high priority.
26. (A) This is the only option that expresses the needed contrast between the previous sentence and the current one since there is a contrast between the glorious construction of the Parthenon and the fact that the glory was very short-lived. The other options do not express the needed contrast.
27. (C) ”Could’ve” sounds like “could of,” but it is short for “could have.” The use of the word “of” in this context is therefore in-correct, making choices (A) and (B) wrong. Choice (C) correctly expresses the verb “have.” Choice (D) makes the sentence say “should have been still served,” which is nonsensical.
28. (D) ”Its” correctly refers to the singular Parthenon’s possession of “pagan gods.” The word “Its’” is always incorrect (choice (A)). Both choice (B) and choice (C) mean “it is.”
29. (D) ”Remnants” means “surviving pieces or traces of something,” so logically these refer to the parts of the Parthenon that remained after its destruction. Choice (A) re-fers to the study of such remains, not the remains themselves. Choices (B) and (C) do not give a precise description of what these are.
30. (D) This choice concisely expresses the intended idea. Choices (A) and (B) are too wordy. Choice (C) likely wants to say something along the lines of “contemporary,” but the word given actually means “with contempt.”
31. (B) ”Beauty” and “refinement” create a compound subject, which re-quires the plural “are.” In addition, the paragraph is in the present tense, so the verb must be in the present tense. Choices (C) and (D) are in the past tense, and choice (A) is singular.
32. (C) The fact that these engravings have lasted for a long time is an essential part of their descrip-tion, so “that” is needed instead of “which.” Choice (C) also uses the proper tense. Choice (A) uses the incorrect verb tense. Choices (B) and (D) use “which,” which works for nonessential characteristics of described objects.
33. (A) The essay focuses throughout on the impressive feat of the Parthenon’s construction, so choice (A) gives a direct connection to this general theme. Choices (B) and (C) are too vague. Choice (D) focuses on only a small part of the passage.
34. (C) By examining the topic sentences of the paragraphs, you can see that the essay is presenting various theories about what happened to the Neanderthals. Choice (C) is therefore the most fitting option to introduce the essay’s argument. Choice (A) is vague, and choice (B) is discon-nected from the essay’s argument. Choice (D) contradicts the essay’s argument since the Neanderthals were not ultimately successful in adaptating.
35. (B) Mention of the Neanderthals at the beginning of the underlined portion is necessary to make a logical comparison with “our own ancestors.” Choices (A) and (D) make illogical comparisons since they compare geographic regions to ancestors. Choice (C) has confusing word order at the end, placing “icebound” such that it literally means that the Neanderthals were icebound. Choice (B) puts things in a logical order and makes a logical comparison of people to Neanderthals.
36. (C) It is necessary to have the words in the order “extreme climate” to express the correct meaning. Since the words must be in this order, no comma is needed to separate them. If the adjectives can be reversed, then a comma between them is necessary (e.g., “the big, tall mountain”).
37. (A) The last part of this sentence gives more support to the claim in the first part of the sentence, making “and” appropriate. Choice (B) shows contrast, choice (C) is too wordy, and choice (D) removes a needed transition.
38. (D) This sentence is best placed at the end of the paragraph since it has the initial transition “further,” which indicates that it is building on the previous argument. Moreover, it is logical to have this after sentence 3 since this sentence gives information in support of the idea that Neanderthals had excellent body heat-generating ability. The clothing cited in the sentence builds on this genetic advantage. Choice (A) makes no sense because this sentence cannot function as an introduction. Choice (B) interrupts a logical transition between sentences 1 and 2. Choice (C) inverts the logical sequence of the inserted sentence building upon sentence 3.
39. (B) The first sentence of the paragraph states “absorption” may have been the cause of Neanderthal extinction. Therefore, “interbreeding” most logically expresses how this absorption could have taken place. Choice (A) would have resulted in Neanderthal extermi-nation. Choices (C) and (D) do not give the strong explanation that “interbreeding” would.
40. (A) ”Contemporary” is the only option that clarifies that these are present-day hu-mans to whom the Neanderthals are compared.
41. (D) This choice concisely expresses the idea using logical word order. Choices (A) and (B) use pas-sive voice. Choice (C) jumbles the word order such that the meaning is confused.
42. (D) Without a clarification of the pronoun, it could be referring to Homo neanderthalensis or to Homo sapiens. Therefore, choices (A) and (B) are too vague. This took place in the past, so choice (D) is correct. The present tense in choice (C) is wrong.
43. (B) ”Margins” is the most logical wording, since Neanderthals would have been pushed to the outer reaches of these geographic areas. Choice (A) does not make sense since there are not oceans in Europe and Asia. Choices (C) and (D) do not make sense since human-like species could not live on “debris” or “remains.”
44. (C) This choice correctly places the clarifying phrase, “nearly back into Africa itself,” out of the way using commas. Choice (A) breaks up the phrase “ancestors first emerged.” Choice (B) breaks up the phrase “edge of the land.” Choice (D) breaks up the phrase “nearly back to Africa.”
Section 3: Math Test (No Calculator)
For some of the problems, an alternative solution, indicated by two asterisks (**) follows the first solution. When this occurs, one of the solutions is the direct mathematical one and the other is based on one of the tactics discussed in Chapters 5 and 6.
1. (A) If John withdraws d dollars every month, the total amount he withdraws in m months is md dollars, and the amount remaining in the account is A – md dollars.
**Plug in easy-to-use numbers. Assume John’s initial deposit is $100 and that he withdraws $10 a month. After 6 months, he will have withdrawn $60 and still have $40 in the account. Which answer choice is equal to 40 when A = 100, d = 10, and m = 6? Only choice (A) works.
2. (D) f(x) < 25 ⇒ x2 − 11 < 25 ⇒ x2 < 36 ⇒ −6 < x < 6
3. (C) Since S represents the number of small pizzas sold during that 6-hour period and L represents the number of large pizzas sold during that same period, S + L is the total number of pizzas sold. So S + L must equal 100. Since each small pizza costs 7.5 dollars and each large pizza costs 11 dollars, 7.5S + 11L is the total number of dollars Joe took in. So this expression must equal 848. The two equations are S + L = 100 and 7.5S + 11L = 848.
4. (D) Note that the left side and the right side of the given equation are equivalent:
3(5 – 2x) = 15 – 6xand6(2 – x) + 3 = 12 – 6x + 3 = 15 – 6x
Since every real number is a solution of the equation 15 – 6x = 15 – 6x, the equation has infinitely many solutions.
**A solution to the equation 3(5 – 2x) = 6(2 – x) + 3 would be the x-coordinate of the point of intersection of the straight lines y = 3(5 – 2x) and y = 6(2 – x) + 3. Since these lines are the same line (they both have the equation y = 15 – 6x), every point on one line is a point on the other.
5. (B) The percent increase in a quantity is 100% (KEY FACT C5). For each year calculate the actual increase and divide. For example, in 2005 the increase wa $100 (from $150 to $250), so the percent increase was . In 2006 the increase was . Check the other choices; this is the greatest.
6. (B)
Any two lines with slopes that are different intersect. Here, the slopes are 1 and –1, so the lines do intersect, and a and b are not necessarily equal. (I is false.)
If a = b, then both lines pass through the point (0, a), which is on the y-axis. (II is true.)
Since a ≠ 0, the point (0, a) dose not lie on the x-axis. (III is false.)
Only statement II is true.
7. (A)
**Use TACTIC 6: Plug in a number for x. For example, if x = 3:
Only choice (A) is 10 when x = 3:
8. (A) y = f(x) + 3 = mx + b + 3 and y = f(x + 3) = m(x + 3) + b = mx + 3m + b. So both graphs are straight lines whose slopes are m. Therefore, the graphs are either a pair of parallel lines (if their y-intercepts are different) or the same line (if their y-intercepts are equal). The y-intercepts are b + 3 and 3m + b, which are equal only if 3m = 3. However, it is given that m ≠ 1, so the intercepts are not equal. The lines are parallel, so the graphs do not intersect.
9. (B) The function is undefined whenever the expression under the square root sign is negative and whenever the denominator is equal to 0.
x – 3 < 0 whenever x < 3. There are 2 positive integers that satisfy this inequal-ity: 1 and 2.
Since x2 – 8x – 20 = (x – 10)(x + 2), this expression is equal to 0 only when x = 10 and x = –2. So there is only 1 positive integer that makes the denominator 0.
In total, there are 3 positive integers for which f(x) has no real values.
10. (A) If 9 = 5 – x, then x = –4. So h(9) = h(5 – (–4)) = (–4)2 + (–4) + 1 = 16 – 4 +1 = 13.
11. (C) Since the volume of the white cube is 27 cubic inches, each edge is 3 inches. Then the area of each face is 9, and the total surface of the cube is 6 × 9 = 54. Each face has a red circle whose ra-dius is 1, so the area of each circle is π(12) = π. Finally, the total red area is 6π, and the total surface area that is NOT red is 54 − 6π.
12. (D) In questions such as these, first find the midpoint of the eligible values. Here, a value for y is acceptable only if 13 ≤ y ≤ 19; the midpoint of this interval is 16. All of the accept-able ages are within 3 years of 16––anywhere from 3 years less than 16 to 3 years greater than 16. The inequality that expresses this is |y – 16| = 3.
13. (B)
Since triangle ABC is equilateral, BC = AB = 12, and so EC = 12 – 8 = 4.
Since is perpendicular to , m∠E = 90°. Since triangle ABC is equilateral, m∠C = 60°. So triangle DEC is a 30-60-90 triangle and .
Since the formula for the area of an equilateral triangle is , the area of triangle ABC is .
The area of triangle DEC is .
So the area of quadrilateral .
14. (B) It should be clear that the answer does not depend on d. So the easiest thing to do, as in all problems involving percents, is to assume that the list price of the book is $100. Then Anne paid $90 after receiving a $10 discount ($10 being 10% of $100). Beth, on the other hand, re-ceived a $15 discount. So she paid $85 for her copy of the book plus a sales tax of 5% of $85, which is 0.05 × $85 = $4.25. So Beth’s total cost was $89.25. Anne paid 75 cents more than Beth.
15. (B) The slope of the line whose equation is y = ax + b is a. To find the slope of the line whose equation is x = cy + d, first solve for y:
Since the slope of the line is the coefficient of x, the slope is . Since parallel lines have equal slopes, .
16. 2 The centers of the two circles are the midpoints of the two diameters.
The midpoint of is .
The midpoint of is .
The slope of the line that passes through (3, 2) and (5, 6) is = .
17. 1 a(x – y) = b(y – x) ⇒ ax – ay = by – bx ⇒ ax + bx = by + ay ⇒ x(a + b) = y(a + b) ⇒ x = y. So the ratio .
18. 2 If 1 + i is a solution of the equation x2 – 2x + c = 0, then
So – 2 + c = 0, and c = 2.
19. 14 Replacing y by x in the equation y = x2 – 6x + 12 gives:
x = x2 – 6x + 12 ⇒ x2 – 7x + 12 = 0 ⇒
(x – 3)(x – 4) = 0 ⇒ x = 3 or x = 4
Since y = x, (a, b) = (3, 3) and (c, d) = (4, 4). So a + b + c + d = 14.
20. 3 If 3 = 5 – 2x, then 2x = 2 and x = 1.
So .
Section 4: Math Test (Calculator)
For some of the problems, an alternative solution, indicated by two asterisks (**) follows the first solution. When this occurs, one of the solutions is the direct mathematical one and the other is based on one of the tactics discussed in Chapters 5 and 6.
1. (B) Wally produces 80 widgets per day × 20 days per month × 12 months per year = 19,200 wid-gets per year; 96,000 ÷ 19,200 = 5.
2. (C) If the total surface area of the cube is 216, then the area of each of the 6 faces is 216 ÷ 6 = 36. Since each face is a square of area 36, each edge is 6. Finally, the volume of the cube is 63 = 216.
3. (A) 2 – 3n ≥ 5 ⇒ –3n ≥ 3. Dividing both sides of this inequality by –3, and remembering to reverse the direction of the inequality gives n ≤ –1. Therefore, 3n ≤ –3, and 2 + 3n ≤ 2 + (–3) ≤ –1.
4. (A) If , then 2x2 – 3 = –2(5 – x2) = –10 + 2x2. Subtracting 2x2 from both sides of this equation gives –3 = –10, which of course is false. So the equation has no solutions.
5. (D) g(–10) = f(3(–10)) = f(–30) = (–30)2 – 3(–30) = 900 – (–90) = 900 + 90 = 990 **g(x) = f(3x) = (3x)2 – 3(3x) = 9x2 – 9x
Then g(–10) = 9(–10)2 – 9(–10) = 900 + 90 = 990.
6. (C) The average is just the sum of the number of students in the five classes (125) divided by 5: 125 ÷ 5 = 25.
7. (A) In class A, one-fourth, or 25% (5 of 20), of the students are in the band. In each of the other classes, the number in the band is less than one-fourth of the class.
8. (D) Let x be the amount, in dollars, that each of the 20 children was going to contribute, then 20x represents the cost of the present. When 4 children dropped out, the remaining 16 each had to pay (x + 1.50) dollars, so
16(x + 1.5) = 20x ⇒ 16x + 24 = 20x ⇒ 24 = 4x ⇒ x = 6
So the cost of the present was 20 × 6 = 120 dollars.
**Use TACTIC 5: backsolve. Try choice (C), 100. If the present cost $100, then each of the 20 chil-dren would have to pay $5. When 4 dropped out, the remaining 16 would have to pay $100 ÷ 16 = $6.25 apiece, an increase of $1.25. Since the actual increase was $1.50, the gift was more expen-sive. Eliminate (A), (B), and (C). The answer must be (D).
9. (A) Since y = –x2, x2 = –y. Replacing x2 by –y in the equation of the circle, we get:
–y + y2 = 90 ⇒ y2 – y – 90 = 0 ⇒ (y – 10)(y + 9) = 0 ⇒ y = 10 or y = –9
A quick glance at the graphs shows that y cannot possibly be equal to 10, so y must equal –9. If you didn’t think to check the graphs, plugging in 10 for y into either equation leads to a contradiction. Since x2 can’t be negative, – x2 can’t be positive. Also if x2 + 102 = 90, then x2 would be negative.
Then x2 + (–9)2 = 90 ⇒ x2 + 81 = 90 ⇒ x2 = 9 ⇒ x = 3 or x = –3. So (a, b) = (–3, –9) and (c, d) = (3, –9).
Finally, a + b + c + d = –3 + –9 + 3 + –9 = –18.
10. (D) Since triangle ABC is a right triangle, we can use the Pythagorean theorem to find BC: 62 + (BC)2 = 102 ⇒ (BC)2 = 100 – 36 = 64 ⇒ BC = 8.
(Of course, if you immediately realize that triangle ABC is a 6-8-10 right triangle, then you don’t have to use the Pythagorean theorem.)
So cos .
11. (D) Let c represent the cost, in dollars, of the candy. Then 3c is the cost of the flowers, and 10(c + 3c) = 10(4c) = 40c is the cost of the ring. So,
528 = 40c + 3c + c = 44c ⇒ c = 528 ÷ 44 = 12
Therefore, the candy cost $12, the flowers cost $36, and the ring cost $480.
12. (D) It should be clear from the diagram that gear G2 turns in a counterclockwise direction. As tooth A pushes against tooth C followed by tooth B pushing against tooth D, the teeth on gear G2 are turning in the opposite direction as those on gear G1.
It should also be clear that the fewer the teeth on a gear, the faster it goes. In fact, the number of teeth is inversely proportional to the number of rotations per second:
number of teeth on G1 × number of rotations per second of G1 =
number of teeth on G2 × number of rotations per second of G2
So if r represents the number of rotations per second of G2, we have
48 × 60 = 36r ⇒ 2,880 = 36r ⇒ r = 2,880 ÷ 36 = 80
13. (C) The difference between the total ticket sales ($35.5 billion) and the total distribution of the pro-ceeds $12.0 billion) was the amount returned to the purchasers of lottery tickets in the form of prize money: $35.5 billion – $12.0 billion = $23.5 billion. Divide 23.5 by 35.5 to see that approxi-mately 66% of the ticket sales was allocated to prize money.
14. (C) In order for the amount received by Senior citizens programs to be the same as the amount re-ceived by the Cities, an additional $0.9 billion would have to be allocated to the Senior citizen pro-grams: $0.9 billion is approximately 31% of the $2.9 billion currently going to the General fund.
15. (D) in 2009, the total sales for all games other than Lotto in billions of dollars was 35.5 – 10.0 = 25.5. In 2010, each of the remaining games experienced a 10% increase in sales, so the total sales in 2010 was 25.5 + 10%(25.5) = 25.5 + 2.55 = 28.05. In 2010, the sales of the Instant games was 10% higher than in 2009: 14.2 + 1.42 = 15.62. So in 2010, the percent of the total sales attributed to Instant games was . Finally, the measure of the central angle for the sector representing Instant games is 55.7% of 360° = 200°.
16. (C) Alice’s gross pay in dollars is:
40 × 9 + (x – 40) × 15 = 360 + 15x – 600 = 15x – 240
23% of her gross pay (8% for payroll taxes and 15% for withholding taxes) is deducted from her gross pay. So her net pay is 77% of her gross pay (100% – 23%) minus a $20 contribution for her health insurance premium:
0.77(15x – 240) – 20
17. (A) (1 + i)3 = [(1 + i)(1 + i)](1 + i) = [1 + 2i + i2](1 + i) = [1 + 2i – 1](1 + i) = [2i](1 + i) = 2i + 2i2 = 2i – 2 = –2 + 2i
18. (D) If the graph of a function crosses the x-axis at n, then (n, 0) is a point on the graph. So (a, 0) and (b, 0) are points on the graph. Therefore, a and b are the solutions of the equation 4x2 – 8x + 3 = 0. There are a few ways to solve this equation.
First solution: Factor 4x2 – 8x + 3 = (2x – 3)(2x – 1). So,
So a and b are and , and .
Second solution: Use the quadratic formula on the equation 4x2 – 8x + 3 = 0.
So or .
Third solution: Use a graphing calculator. Graph y = 4x2 – 8x + 3, and see where the graph crosses the x-axis.
19. (C) The central angle of the sector representing Brand C is 12% of 360°:
(0.12) × 360° = 43.2°
The central angle of the sector representing Brand D is 7% of 360°:
(0.7) × 360° = 25.2°
Finally, 43.2° – 25.2° = 18°.
**Note this can be done in one step by noticing that the percentage difference between Brands C and D is 5%, and 5% of 360 is (0.05) × 360 = 18.
20. (C) Since total sales in 2000 were $1,000,000, in 2005 sales were $1,500,000 (a 50% increase).
In 2000, sales of Brand A were $400,000 (40% of $1,000,000).
In 2005, sales of Brand A were $500,000 (25% or more than in 2000).
Finally, $500,000 is or of $1,500,000.
21. (D) Store 2 sold 30 DVD players at $50, 40 at $80, and 20 at $120.
Store 2: (30 × $50) + (40 × $80) + (20 × $120) = $7,100
Store 1 sold 10 DVD players at $80, 20 at $100, and 20 at $150.
Store 1: (10 × $80) + (20 × $100) + (20 × $150) = $5,800
Finally, $7,100 – $5,800 = $1,300.
22. (B) The face of the solid is an isosceles triangle whose base is 16. To find its area, draw in altitude in the diagram below.
Since the altitude to the base of an isosceles triangle is also a median, AD = DC = 8. So tri-angle ABD is a 6-8-10 right triangle. (If you don’t immediately recognize that, use the Pythagorean theorem: 102 = 82 + (BD)2 ⇒ (BD)2 = 100 – 64 = 36 ⇒ BD = 6.) The area of triangle ABC is
23. (B) The solid is a prism whose triangular face has an area of 48 square feet (see the solution to question 22) and whose width is w. So the volume of the solid is 48w cubic feet, and its weight is
(48w cubic feet) × (3 pounds per cubic foot) = 144w pounds
Then 144w = 360 ⇒ w = 360 ÷ 144 = 2.5.
24. (D) Since they have the same angles, by KEY FACT J16 all equilateral triangles are similar. If k is the ratio of the sides of two similar triangles, then by KEY FACT J18 the ratio of their areas is k2. Here the ratio of the areas is 6:10, or 0.6, so k2 = 0.6 and .
25. (C) Here are two ways to answer this question.
You can just put your calculator into radian mode and evaluate:
Leave your calculator in degree mode and convert radians to degrees: radians = = 60 degrees
(sin 60° + cos 60°)2 = (0.866 + 0.5)2 = 1.3662 = 1.866
26. (D) For each of the given years, the total acreage of farmland can be calculated by multiplying the number of farms by the average size of a farm. There is no way to answer this question, without do-ing the calculation for each year.
1940: 6.3 million farms × 174 acres per farm = 1,096 million acres
1970: 2.9 million farms × 374 acres per farm = 1,085 million acres
2000: 2.2 million farms × 436 acres per farm = 959 million acres
2010: 2.2 million farms × 418 acres per farm = 919 million acres
27. (C) Find the sum: 130 + 61 + 46 + 46 + 43 = 326. The sum of the farm acreages for the five states is 326 million acres. In the solution to the previous question, we saw that the total farm acreage in the country in 2010 was 919 million acres, and 326 ÷ 919 = 0.3547 or approximately 35%.
28. (B) It is projected that from 2010 to 2030, the number of farms will decrease by 5%. Since 5% of 2.2 million is 110,000, the number of farms in 2030 will be approximately 2,200,000 – 110,000 = 2,090,000 = 2.09 million. From 1990 to 2010, the average size of a farm decreased by 42 acres from 460 acres to 418 acres, and 42 ÷ 460 = 0.091 = 9.1%. If from 2010 to 2030 the average size of a farm again decreases by 9.1%, there will be a decrease of 0.091 × 418 = 38 acres, bringing the average size to 418 – 38 = 380 acres. Finally, the total farm acreage in 2030 in millions of acres is projected to be about 2.09 × 380 = 794 ≈ 800.
29. (C) The method we use to convert the given equation, x2 + 2x + y2 – 4y + 1 = 0, into the standard form for a circle, (x – h)2 + (y – k)2 = r2, is completing the square (see KEY FACT N10).
So the center of the circle, (h, k), is the point (–1, 2) and the radius, r, is 2. Finally, h + k + r = –1 + 2 + 2 = 3.
30. (C) The formula for the area of a parallelogram is A = bh. Sketch the parallelogram and draw the height.
To find h, use the sine ratio:
sin sin 45° = 10(0.707) = 7.07
An alternative method of finding h is to use the fact that in a 45-45-90 right triangle, the length of each leg is equal to the length of the hypotenuse divided by .
Either way, h = 7.07 and the area of the parallelogram is:
A = bh = 10(7.07) = 70.7
31. 37.5 First set up a proportion to determine the distance between Hauppauge and Riverhead:
So x = (1.25)(20) = 25. Traveling at 40 miles per hour, it takes of an hour to drive 25 miles. Finally, of an hour is equal to (60 minutes) = 37.5 minutes.
32. 6.8 Since Michelle took 31.5 minutes to run the 3 miles from the 12-mile marker to the 15-mile marker, she was running at the constant rate of 1 mile every 10.5 minutes (31.5 ÷ 3 = 10.5). Michelle took 54.6 minutes (36 seconds = 36 ÷ 60 = 0.6 minutes) to run from where her friend was standing to the 12-mile marker. So that distance was 54.6 minutes ÷ 10.5 minutes per mile = 5.2 miles.
Her friend was 5.2 miles from the 12-mile marker, and so was 12 – 5.2 = 6.8 miles from the starting line.
33. 3744 Use a weighted average:
34. 8190 There are 26 × 26 × 9 = 6,084 PICs with two letters and one digit, and there are 26 × 9 × 9 = 2,106 PICs with one letter and two digits, for a total of 6,084 + 2,106 = 8,190.
35. or 1.51 The formula for the volume of a pyramid with a rectangular base is , where l and w are the length and width of the rectangle, and where h is the height of the pyramid. (Remember that this fact is given to you on the first page of each math sec-tion.)
The base of pyramid 1 is a 2 by 3 rectangle whose area is (2)(3) = 6.
The base of pyramid 2 is a square of side 3 whose area is 32 = 9. If h1 and h2 represent the two heights, then:
36. 129 Let c, d, and e represent the value of one Canadian dollar, one U.S. dollar, and one euro, respectively. Since c = .68e, we have that . Then we have:
So, one U.S. dollar could have purchased 1.294 Canadian dollars, and 100 U.S. dollars could have purchased 129.40, or approximately 129 Canadian dollars.
37. 54
60 miles per hour = 60 miles per 60 minutes = 1 mile per 1 minute. Driving at 60 miles per hour, Ed takes 6.3 minutes to drive those 6.3 miles.
70 miles per hour = 70 miles per 60 minutes = mile per 1 minute. Driving at 70 miles per hour, Ed would take minutes to drive those 6.3 miles.
So Ed would take 6.3 – 5.4 = 0.9 minutes less to drive those 6.3 miles at 70 miles per hour.
Finally, 0.9 minutes = 0.9 × 60 seconds = 54 seconds.
38. 37
6.3 miles per day × 240 days = 1,512 miles.
At 60 miles per hour, Ed used 1,512 ÷ 24.3 = 62.222 gallons of gasoline, which cost 62.222 × $3.50 = $217.78.
At 70 miles per hour, Ed would have used 1,512 ÷ 20.8 = 72.692 gallons of gasoline, which would have cost 72.692 × $3.50 = $254.42.
So traveling at 70 miles per hour instead of 60 miles per hour would have cost Ed $254.42 – $217.78 = $36.64 more, which to the nearest dollar is $37.
SAT Essay Scoring
SAT Essay Scoring Rubric
Score: 4 | |
Reading | Excellent: The essay shows excellent understanding of the source.
The essay shows an understanding of the source’s main argument and key details and a firm grasp of how they are interconnected, demonstrating clear comprehension of the source. The essay does not misinterpret or misrepresent the source. The essay skillfully uses source evidence, such as direct quotations and rephrasing, representing a thorough comprehension of the source. |
Analysis | Excellent: The essay gives excellent analysis of the source and shows clear understanding of what the assignment requires.
The essay gives a complete, highly thoughtful analysis of the author’s use of reasoning, evidence, rhetoric, and/or other argumentative elements the student has chosen to highlight. The essay has appropriate, adequate, and skillfully chosen support for its analysis. The essay focuses on the most important parts of the source in responding to the prompt. |
Writing | Excellent: The essay is focused and shows an excellent grasp of the English language. The essay has a clear thesis. The essay has a well-executed introduction and conclusion. The essay shows a clear and well-crafted progression of thoughts both within paragraphs and in the essay as a whole. The essay has a wide range of sentence structures. The essay consistently shows precise choice of words. The essay is formal and objective in its style and tone. The essay demonstrates a firm grasp of the rules of standard English and has very few to no errors. |
Score: 3 | |
Reading | Skillful: The essay shows effective understanding of the source.
The essay shows an understanding of the source’s main argument and key details. The essay is free of major misinterpretations and/or misrepresentations of the source. The essay uses appropriate source evidence, such as direct quotations and rephrasing, representing comprehension of the source. |
Analysis | Skillful: The essay gives effective analysis of the source and shows an understanding of what the assignment requires.
The essay decently analyzes the author’s use of reasoning, evidence, rhetoric, and/or other argumentative elements the student has chosen to highlight. The essay has appropriate and adequate support for its analysis. The essay focuses primarily on the most important parts of the source in responding to the prompt. |
Writing | Skillful: The essay is mostly focused and shows an effective grasp of the English language. The essay has a thesis, either explicit or implicit. The essay has an effective introduction and conclusion. The essay has a clear progression of thoughts both within paragraphs and in the essay as a whole. The essay has an assortment of sentence structures. The essay shows some precise choice of words. The essay is formal and objective in its style and tone. The essay demonstrates a grasp of the rules of standard English and has very few significant errors that interfere with the writer’s argument. |
Score: 2 | |
Reading | Limited: The essay shows some understanding of the source.
The essay shows an understanding of the source’s main argument, but not of key details. The essay may have some misinterpretations and/or misrepresentations of the source. The essay gives only partial evidence from the source, showing limited comprehension of the source. |
Analysis | Limited: The essay gives partial analysis of the source and shows only limited understanding of what the assignment requires.
The essay tries to show how the author uses reasoning, evidence, rhetoric, and/or other argumentative elements the student has chosen to highlight, but only states rather than analyzes their importance, or at least one part of the essay’s analysis is unsupported by the source. The essay has little or no justification for its argument. The essay may lack attention to those elements of the source that are most pertinent to responding to the prompt. |
Writing | Limited: The essay is mostly not cohesive and shows an ineffective grasp of the English language. The essay may not have a thesis, or may diverge from the thesis at some point in the essay’s development. The essay may have an unsuccessful introduction and/or conclusion. The essay may show progression of thoughts within the paragraphs, but not in the essay as a whole. The essay is relatively uniform in its sentence structures. The essay shows imprecise and possibly repetitive choice of words.
The essay may be more casual and subjective in style and tone. The essay demonstrates a weaker grasp of the rules of standard English and does have errors that interfere with the writer’s argument. |
Score: 1 | |
Reading | Insufficient: The essay shows virtually no understanding of the source.
The essay is unsuccessful in showing an understanding of the source’s main argument. It may refer to some details from the text, but it does so without tying them to the source’s main argument. The essay has many misinterpretations and/or misrepresentations of the source. The essay gives virtually no evidence from the source, showing very poor comprehension of the source. |
Analysis | Insufficient: The essay gives little to no accurate analysis of the source and shows poor understanding of what the assignment requires.
The essay may show how the author uses reasoning, evidence, rhetoric, and/or other argumentative elements that the student has chosen to highlight but does so without analysis. Or many parts of the essay’s analysis are unsupported by the source. The support given for points in the essay’s argument are largely unsupported or off topic. The essay may not attend to the elements of the source that are pertinent to responding to the prompt. Or the essay gives no explicit analysis, perhaps only resorting to summary statements. |
Writing | Insufficient: The essay is not cohesive and does not demonstrate skill in the English language. The essay may not have a thesis. The essay does not have a clear introduction and conclusion. The essay does not have a clear progression of thoughts. The essay is quite uniform and even repetitive in sentence structure.
The essay shows poor and possibly inaccurate word choice. The essay is likely casual and subjective in style and tone. The essay shows a poor grasp of the rules of standard English and may have many errors that interfere with the writer’s argument. |
Top-Scoring Sample Student Response
In the passage, “A Lesson on Commas,” the author is presented with an off-topic question from one of her students in regards to why she chooses to write. When faced with a crossroads on whether to continue on with her planned lesson, or go off on a tangent and respond to the student’s question, the teacher uses the opportunity to explain why creativity must be taught in the classroom.
As the author looks around the room and notices that her own lesson may be lacking in creativity, specifically in regards to “the abridged somnolence” of the students “in the farthest row,” she realizes that she has an opportunity to seize the moment and further explain why this is so important. She first looks to begin her argument by engaging the students with a creative response—Why does one listen to music? Or dance? Or look to the stars? And then goes on to state her belief in regards to the importance of creativity in that it is “the most essential ingredient of erudition, expression, and future success,” in order to make an impactful statement to catch her students’ attention.
Her next focus is to debunk the concept that the term “creativity” is only applicable to art. To support her belief that creativity “is not reserved for the Picassos, the Bachs, or the Austens,” she identifies that it is something that is rooted within all of us, which she bolsters by asking the class to think about the “sublime imagination” of any child that they have ever met. Furthermore, she identifies that creativity is used in every walk of life, as it is the primary driver behind not just art, but also “every new electronic, and every scientific discovery.” Statements such as these help to connect students that might not see themselves as “artistic” to the fact that “creativity” is still relevant to them.
Mrs. Jenson then explains the significance that creativity holds, which she does by asking her students a simple question about their future. While alluding to the idea that everyone would prefer a “challenging” job over a “routine” one, she quotes a well-known source, LinkedIn, on the fact that “not being challenged . . . is at the top of the list for workplace dissatisfaction” in order to add validity to her argument. When tied together with the fact that “hating their jobs” is the “number one reason for unhappiness in adults,” she is able to make the greater connection between how one’s work-life impacts their overall well being.
However, this alone does not explain how creativity directly ties in with feeling challenged at the workplace. While acknowledging that it is easier to see how creativity is used by some of the world’s top minds, in fields such as medicine and engineering, she uses a two-step process in order to bring this together for those that may end up in less notable areas. The first is by referencing creativity in a “Google” sense, undoubtedly a concept that students are familiar with. By breaking down the meaning of creativity to its most general terms, “generation and contemplation of new ideas to create something perceived as valuable,” we can think of it as something that can be used in almost any setting. And when applied in the context of a working environment, creativity can therefore be thought of in terms of our “problem-solving and critical thinking” abilities, which in turn helps with our “efficiency, diversification, and job security.” In this sense, she is explaining how one must use creativity in order to help make their job more challenging, which will in turn help improve their overall happiness in their adult life.
Having stated her belief in the importance of creativity, providing support for how it is relevant for everyone to practice, and making the connection between it, work, and a healthy sense of self-worth, the author then addresses how this can be taught in school. Whether it be by “encouraging questioning and debate” to get students to think about “the why’s and how’s,” “identifying problems and brainstorming solutions” to help students determine “what’s good, what’s bad and what can be improved,” or by “inspiring alternative problem-solving” to enable students to consider the “what-ifs,” she identifies many ways to help foster creativity. And because of her aforementioned statements regarding the importance of creative thinking, her conclusion serves to make a sound argument for the importance of actively teaching creativity in schools.